Вы находитесь на странице: 1из 190

Problems of the 20th International Physics Olympiad 1

(Warsaw, 1989)
Waldemar Gorzkowski
Institute of Physics, Polish Academy of Sciences, Warsaw, Poland 2
Abstract
The article contains problems given at the 20th International Physics Olympiad (1989)
and their solutions. The 20th IPhO was the third IPhO organized in Warsaw, Poland.
Logo
The emblem of the XX International Physics
Olympiad contains a picture that is a historical record of
the first hypernuclear event observed and interpreted in
Warsaw by M. Danysz and J. Pniewski 3. The collision
of a high-energy particle with a heavy nucleus was
registered in nuclear emulsion. Tracks of the secondary
particles emitted in the event, seen in the picture (upper
star), consist of tracks due to fast pions (thin tracks)
and to much slower fragments of the target nucleus
(black tracks). The black track connecting the upper
star (greater) with the lower star (smaller) in the figure
is due to a hypernuclear fragment, in this case due to a
part of the primary nucleus containing an unstable
hyperon instead of a nucleon. Hyperfragments
(hypernuclei) are a new kind of matter in which the nuclei contain not only protons and
neutrons but also some other heavy particles.
In the event observed above the hyperon , bound with nucleon, decays like a free
particle through a week (slow) process only. This fact strongly suggested the existence of a
new quantum number that could explain suppression of the decay, even in presence of
nucleons. Indeed, this was one of the observations that, 30 months later, led to the concept of
strangeness.
Introduction
Theoretical problems (including solutions and marking schemes) were prepared
especially for the 20th IPhO by Waldemar Gorzkowski. The experimental problem (including
the solution and marking scheme) was prepared especially for this Olympiad by Andrzej
Kotlicki. The problems were refereed independently (and many times) by at least two persons
1

This article has been sent for publication in Physics Competitions in October 2003
e-mail: gorzk@ifpan.edu.pl
3
M. Danysz and J. Pniewski, Bull. Acad. Polon. Sci., 3(1) 42 (1952) and Phil. Mag., 44, 348 (1953). Later the
same physicists, Danysz and Pniewski, discovered the first case of a nucleus with two hyperons (double
hyperfragment).
2

after any change was made in the text to avoid unexpected difficulties at the competition. This
work was done by:
First Problem:
Andrzej Szadkowski, Andrzej Szymacha, Wodzimierz Ungier
Second Problem:
Andrzej Szadkowski, Andrzej Szymacha, Wodzimierz Ungier, Stanisaw Woronowicz
Third Problem:
Andrzej Rajca, Andrzej Szymacha, Wodzimierz Ungier
Experimental Problem:
Krzysztof Korona, Anna Lipniacka, Jerzy usakowski, Bruno Sikora
Several English versions of the texts of the problems were given to the Englishspeaking students. As far as I know it happened for the first time (at present it is typical). The
original English version was accepted (as a version for the students) by the leaders of the
Australian delegation only. The other English-speaking delegations translated the English
originals into English used in their countries. The net result was that there were at least four
English versions. Of course, physics contained in them was exactly the same, while wording
and spelling were somewhat different (the difference, however, were not too great).
This article is based on the materials quoted at the end of the article and on personal
notes of the author.
THEORETICAL PROBLEMS
Problem 1
Consider two liquids A and B insoluble in each other. The pressures pi (i = A or B) of
their saturated vapors obey, to a good approximation, the formula:
ln( pi / po ) =

i
T

+ i ;

i = A or B,

where po denotes the normal atmospheric pressure, T the absolute temperature of the vapor,
and i and i (i = A or B) certain constants depending on the liquid. (The symbol ln
denotes the natural logarithm, i.e. logarithm with base e = 2.7182818)
The values of the ratio pi/p0 for the liquids A and B at the temperature 40C and 90C
are given in Tab. 1.1.
Table 1.1
t [C]
40
90

pi/p0
i=A
0.284
1.476

i=B
0.07278
0.6918

The errors of these values are negligible.

A. Determine the boiling temperatures of the liquids A and B under the pressure p0.

B. The liquids A and B were poured into a vessel in which the layers shown in Fig. 1.1
were formed. The surface of the liquid B has been covered with a thin layer of a non-volatile
liquid C, which is insoluble in the liquids A and B and vice versa, thereby preventing any free
evaporation from the upper surface of the liquid B, The ratio of molecular masses of the
liquids A and B (in the gaseous phase) is:

= A / B = 8.
p0

p0

p0

C
t1
B
t2
A

1
Fig. 1.1

Fig. 1.2

The masses of the liquids A and B were initially the same, each equal to m = 100g. The
heights of the layers of the liquids in the vessel and the densities of the liquids are small
enough to make the assumption that the pressure in any point in the vessel is practically equal
to the normal atmospheric pressure p0.
The system of liquids in the vessel is slowly, but continuously and uniformly, heated. It
was established that the temperature t of the liquids changed with time as shown
schematically in the Fig. 1.2.
Determine the temperatures t1 and t2 corresponding to the horizontal parts of the
diagram and the masses of the liquids A and B at the time 1. The temperatures should be
rounded to the nearest degree (in C) and the masses of the liquids should be determined to
one-tenth of gram.
REMARK: Assume that the vapors of the liquids, to a good approximation,
(1) obey the Dalton law stating that the pressure of a mixture of gases is equal to
the sum of the partial pressures of the gases forming the mixture and
(2) can be treated as perfect gases up to the pressures corresponding to the
saturated vapors.
Solution
PART A

The liquid boils when the pressure of its saturated vapor is equal to the external pressure.
Thus, in order to find the boiling temperature of the liquid i (i - A or B), one should determine
such a temperature Tbi (or tbi) for which pi/p0 = 1.
Then ln( pi / p0 ) = 0 , and we have:
Tbi =

i
.
i

The coefficients i and i are not given explicitly. However, they can be calculated
from the formula given in the text of the problem. For this purpose one should make use of
the numerical data given in the Tab. 1.1.
For the liquid A, we have:
ln 0.284 =
ln1.476 =

+ A,

(40 + 273.15)K

A
(90 + 273.15)K

+ A.

After subtraction of these equations, we get:


1
1

-1
ln 0.284 ln1.476 = A

K .
40 + 273.15 90 + 273.15

A =

ln

0.284
1.476

1
1

40 + 273.15 90 + 273.15

K -3748.49K .

Hence,

A = ln 0.284

A
(40 + 273.15)K

10.711 .

Thus, the boiling temperature of the liquid A is equal to


TbA = 3748.49K/10.711 349.95 K.
In the Celsius scale the boiling temperature of the liquid A is
t bA = (349.95 273.15)C = 76.80C 77C.
For the liquid B, in the same way, we obtain:

B -5121.64 K,
B 13.735,
TbB 372-89 K,
t bB 99.74C 100C.
PART B

As the liquids are in thermal contact with each other, their temperatures increase in time
in the same way.
At the beginning of the heating, what corresponds to the left sloped part of the diagram,
no evaporation can occur. The free evaporation from the upper surface of the liquid B cannot
occur - it is impossible due to the layer of the non-volatile liquid C. The evaporation from the
inside of the system is considered below.

Let us consider a bubble formed in the liquid A or in the liquid B or on the surface that
separates these liquids. Such a bubble can be formed due to fluctuations or for many other
reasons, which will not be analyzed here.
The bubble can get out of the system only when the pressure inside it equals to the
external pressure p0 (or when it is a little bit higher than p0 ). Otherwise, the bubble will
collapse.
The pressure inside the bubble formed in the volume of the liquid A or in the volume of
the liquid B equals to the pressure of the saturated vapor of the liquid A or B, respectively.
However, the pressure inside the bubble formed on the surface separating the liquids A and B
is equal to the sum of the pressures of the saturated vapors of both these liquids, as then the
bubble is in a contact with the liquids A and B at the same time. In the case considered the
pressure inside the bubble is greater than the pressures of the saturated vapors of each of the
liquids A and B (at the same temperature).
Therefore, when the system is heated, the pressure p0 is reached first in the bubbles that
were formed on the surface separating the liquids. Thus, the temperature t1 corresponds to a
kind of common boiling of both liquids that occurs in the region of their direct contact. The
temperature t1 is for sure lower than the boiling temperatures of the liquids A and B as then
the pressures of the saturated vapors of the liquids A and B are less then p0 (their sum equals
to p0 and each of them is greater than zero).
In order to determine the value of t1 with required accuracy, we can calculate the values
of the sum of the saturated vapors of the liquids A and B for several values of the temperature
t and look when one gets the value p0 .
From the formula given in the text of the problem, we have:

A
+ A
pA
,
=eT
p0

(1)

B
+B
pB
.
=eT
p0

(2)

p A + p B equals to p0 if
p A pB
+
= 1.
p0 p0
Thus, we have to calculate the values of the following function:
A

y ( x) = e

t + t0

+ A

+e

t + t0

+B

(where t 0 = 273.15 C) and to determine the temperature t = t1 , at which y (t ) equals to 1.


When calculating the values of the function y (t ) we can divide the intervals of the
temperatures t by 2 (approximately) and look whether the results are greater or less than 1.
We have:

Table 1.2
y (t )

t
40C

< 1 (see Tab. 1.1)

77C

> 1 (as t1 is less than t bA )

59C

0.749 < 1

70C

1.113 > 1

66C

0.966 < 1

67C

1.001 > 1

66.5C

0.983 < 1

Therefore, t1 67 C (with required accuracy).


Now we calculate the pressures of the saturated vapors of the liquids A and B at the
temperature t1 67C, i.e. the pressures of the saturated vapors of the liquids A and B in each
bubble formed on the surface separating the liquids. From the equations (1) and (2), we get:

p A 0.734 p0 ,
p B 0.267 p0 ,
( p A + p B = 1.001 p0 p0 ) .
These pressures depend only on the temperature and, therefore, they remain constant
during the motion of the bubbles through the liquid B. The volume of the bubbles during this
motion also cannot be changed without violation of the relation p A + p B = p0 . It follows from
the above remarks that the mass ratio of the saturated vapors of the liquids A and B in each
bubble is the same. This conclusion remains valid as long as both liquids are in the system.
After total evaporation of one of the liquids the temperature of the system will increase again
(second sloped part of the diagram). Then, however, the mass of the system remains constant
until the temperature reaches the value t 2 at which the boiling of the liquid (remained in the
vessel) starts. Therefore, the temperature t 2 (the higher horizontal part of the diagram)
corresponds to the boiling of the liquid remained in the vessel.
The mass ratio m A / mB of the saturated vapors of the liquids A and B in each bubble
leaving the system at the temperature t1 is equal to the ratio of the densities of these vapors
A / B . According to the assumption 2, stating that the vapors can be treated as ideal gases,
the last ratio equals to the ratio of the products of the pressures of the saturated vapors by the
molecular masses:
mA A p A A p A
=
=
=
.
mB B p B B p B

Thus,
mA
22.0 .
mB

We see that the liquid A evaporates 22 times faster than the liquid B. The evaporation of
100 g of the liquid A during the surface boiling at the temperature t1 is associated with the

evaporation of 100 g / 22 4.5 g of the liquid B. Thus, at the time 1 the vessel contains 95.5
g of the liquid B (and no liquid A). The temperature t 2 is equal to the boiling temperature of
the liquid B: t 2 = 100C.
Marking Scheme
1.
2.
3.
4.
5.
6.
7.
8.

physical condition for boiling


boiling temperature of the liquid A (numerical value)
boiling temperature of the liquid B (numerical value)
analysis of the phenomena at the temperature t1
numerical value of t1
numerical value of the mass ratio of the saturated vapors in the bubble
masses of the liquids at the time 1
determination of the temperature t 2

1 point
1 point
1 point
3 points
1 point
1 point
1 point
1 point

REMARK: As the sum of the logarithms is not equal to the logarithm of the sum, the
formula given in the text of the problem should not be applied to the mixture of the saturated
vapors in the bubbles formed on the surface separating the liquids. However, the numerical
data have been chosen in such a way that even such incorrect solution of the problem gives
the correct value of the temperature t1 (within required accuracy). The purpose of that was to
allow the pupils to solve the part B of the problem even if they determined the temperature t1
in a wrong way. Of course, one cannot receive any points for an incorrect determination of the
temperature t1 even if its numerical value is correct.
Typical mistakes in the pupils' solutions
Nobody has received the maximum possible number of points for this problem,
although several solutions came close. Only two participants tried to analyze proportion of
pressures of the vapors during the upward movement of the bubble trough the liquid B. Part
of the students confused Celsius degrees with Kelvins. Many participants did not take into
account the boiling on the surface separating the liquids A and B, although this effect was the
essence of the problem. Part of the students, who did notice this effect, assumed a priori that
the liquid with lower boiling temperature "must" be the first to evaporate. In general, this need
not be true: if were, for example, 1/8 instead 8, then liquid A rather than B would remain in
the vessel. As regards the boiling temperatures, practically nobody had any essential
difficulties.
Problem 2
Three non-collinear points P1, P2 and P3, with known masses m1, m2 and m3, interact
with one another through their mutual gravitational forces only; they are isolated in free space
and do not interact with any other bodies. Let denote the axis going through the center-ofmass of the three masses, and perpendicular to the triangle P1P2P3. What conditions should
the angular velocities of the system (around the axis ) and the distances:
P1P2 = a12,

P2P3 = a23,

P1P3 = a13,

fulfill to allow the shape and size of the triangle P1P2P3 unchanged during the motion of the
system, i.e. under what conditions does the system rotate around the axis as a rigid body?

Solution
As the system is isolated, its total energy, i.e. the sum of the kinetic and potential
energies, is conserved. The total potential energy of the points P1, P2 and P3 with the masses
m1 , m2 and m3 in the inertial system (i.e. when there are no inertial forces) is equal to the
sum of the gravitational potential energies of all the pairs of points (P1,P2), (P2,P3) and (P1,P3).
It depends only on the distances a12 , a23 and a23 which are constant in time. Thus, the total
potential energy of the system is constant. As a consequence the kinetic energy of the system
is constant too. The moment of inertia of the system with respect to the axis depends only
on the distances from the points P1, P2 and P3 to the axis which, for fixed a12 , a23 and a23
do not depend on time. This means that the moment of inertia I is constant. Therefore, the
angular velocity of the system must also be constant:

= const.

(1)

This is the first condition we had to find. The other conditions will be determined by
using three methods described below. However, prior to performing calculations, it is
desirable to specify a convenient coordinates system in which the calculations are expected to
be simple.
Let the positions of the points P1, P2 and P3 with the masses m1 , m2 and m3 be given by
the vectors r1 , r2 and r3 . For simplicity we assume that the origin of the coordinate system is
localized at the center of mass of the points P1, P2 and P3 with the masses m1 , m2 and m3 and
that all the vectors r1 , r2 and r3 are in the same coordinate plane, e.g. in the plane (x,y). Then
the axis is the axis z .
In this coordinate system, according to the definition of the center of mass, we have:
m1r1 + m2 r2 + m3r2 = 0

(2)

Now we will find the second condition by using several methods.


FIRST METHOD

Consider the point P1 with the mass m1 . The points P2 and P3 act on it with the forces:
F21 = G

m1m2
(r2 r1 ),
a123

(3)

F31 = G

m1m3
(r3 r1 ).
a133

(4)

where G denotes the gravitational constant.


In the inertial frame the sum of these forces is the centripetal force
Fr1 = m1 2 r1 ,
which causes the movement of the point P1 along a circle with the angular velocity . (The
moment of this force with respect to the axis is equal to zero.) Thus, we have:
F21 + F31 = Fr1 .

(5)

In the non-inertial frame, rotating around the axis with the angular velocity , the
sum of the forces (3), (4) and the centrifugal force

F' r1 = m1 2 r1
should be equal to zero:
F21 + F31 + F' r1 = 0.

(6)

(The moment of this sum with respect to any axis equals to zero.)
The conditions (5) and (6) are equivalent. They give the same vector equality:
G

mm
m1m2
(r2 r1 ) + G 1 3 3 (r3 r1 ) + m1 2 r1 = 0,
3
a12
a13

2 Gm2 Gm3
m1
m1
3 3 = 0
m
G
m
+
m
r
+
r
r
2
2
3
3
1
1
a123
a133
a12
a13

(7)

(7)

From the formula (2), we get:


m2 r2 = m1r1 m3r3

(8)

Using this relation, we write the formula (7) in the following form:

m1
m
Gm Gm
(m1r1 m3r3 ) + G 31 m3r3 + m1r1 2 3 2 3 3 = 0 ,
3
a12
a13
a12
a13

i.e.

1
Gm Gm Gm
1
r1m1 2 3 2 3 3 3 1 + r3 3 3 Gm1m3 = 0.
a12
a13
a12

a13 a12
The vectors r1 and r3 are non-col1inear. Therefore, the coefficients in the last formula
must be equal to zero:

1
1
3 3 Gm1m3 = 0,
a13 a12

Gm Gm Gm
m1 2 3 2 3 3 3 1 = 0.
a12
a13
a12

The first equality leads to:


1
1
= 3
3
a13 a12

and hence,
a13 = a12 .
Let a13 = a12 = a . Then the second equality gives:

2 a 3 = GM

(9)

where
M = m1 + m2 + m3
denotes the total mass of the system.

(10)

In the same way, for the points P2 and P3, one gets the relations:
a) the point P2:
a23 = a12 ;

2 a 3 = GM

a13 = a23 ;

2 a 3 = GM

b) the point P3:

Summarizing, the system can rotate as a rigid body if all the distances between the
masses are equal:
a12 = a23 = a13 = a ,

(11)

the angular velocity is constant and the relation (9) holds.


SECOND METHOD

At the beginning we find the moment of inertia I of the system with respect to the axis

. Using the relation (2), we can write:

0 = (m1r1 + m2 r2 + m3r3 ) 2 = m12 r12 + m22 r22 + m32 r32 + 2m1m2 r1r2 + 2m1m3r1r3 + 2m3 m2 r3r2 .

Of course,
ri2 = ri 2

i = 1, 2, 3

The quantities 2ri r j (i, j = 1, 2, 3) can be determined from the following obvious relation:
2

aij2 = ri r j = (ri r j ) 2 = ri2 + r j2 2ri r j = ri 2 + r j2 2ri r j .


We get:

2ri r j = ri 2 + r j2 aij2 .
With help of this relation, after simple transformations, we obtain:
0 = (m1r1 + m2 r2 + m3r3 ) 2 = (m1 + m2 + m3 )(m1r12 + m2 r22 + m3 r32 ) mi m j aij2 .
i< j

The moment of inertia I of the system with respect to the axis , according to the definition
of this quantity, is equal to
I = m1r12 + m2 r22 + m3 r32 .

The last two formulae lead to the following expression:


I=

1
M

m m a
i< j

2
ij

where M (the total mass of the system) is defined by the formula (10).
In the non-inertial frame, rotating around the axis with the angular velocity , the
total potential energy Vtot is the sum of the gravitational potential energies
Vij = G

mi m j
aij

of all the masses and the potential energies

i, j = 1, 2, 3; i < j

Vi = 12 2 mi ri 2 ;

i = 1, 2, 3

of the masses mi (i = 1, 2, 3) in the field of the centrifugal force:


Vtot = G
i< j

mi m j
aij

mi m j

i =1

aij

12 2 mi ri 2 = G

2 2 G
a +
= mi m j
2 M ij a
i< j
ij

i< j

12 2 I = G

mi m j

i< j

aij

12 2

1
M

m m a
i< j

2
ij

i.e.
2 2 G
Vtot = mi m j
a +
2 M ij a
i< j
ij

A mechanical system is in equilibrium if its total potential energy has an extremum. In


our case the total potential energy Vtot is a sum of three terms. Each of them is proportional to:
f (a) =

2
2M

a2 +

G
.
a

The extrema of this function can be found by taking its derivative with respect to a and
requiring this derivative to be zero. We get:

2
M

G
= 0.
a2

It leads to:

2 a 3 = GM

or

2 a 3 = G (m1 + m2 + m3 ).

We see that all the terms in Vtot have extrema at the same values of aij = a . (In addition,
the values of a and should obey the relation written above.) It is easy to show that it is a
maximum. Thus, the quantity Vtot has a maximum at aij = a .
This means that our three masses can remain in fixed distances only if these distances
are equal to each other:
a12 = a23 = a13 = a
and if the relation

2 a 3 = GM ,
where M the total mass of the system, holds. We have obtained the conditions (9) and (11)
again.
THIRD METHOD

Let us consider again the point P1 with the mass m1 and the forces F21 and F31 given by
the formulae (3) and (4). It follows from the text of the problem that the total moment (with
respect to any fixed point or with respect to the mass center) of the forces acting on the point
P1 must be equal to zero. Thus, we have:
F21 r1 + F31 r1 = 0
where the symbol denotes the vector product. Therefore

mm
m1m2
(r2 r1 ) r1 + G 1 3 3 (r3 r1 ) r1 = 0.
3
a12
a13

But

r1 r1 = 0.
Thus:
m
m2
r2 r1 + 33 r3 r1 = 0.
3
a13
a12
Using the formula (8), the last relation can be written as follows:
m
1
(m1r1 m3r3 ) r1 + 33 r3 r1 = 0,
3
a12
a13

m3
m
r r1 + 33 r3 r1 = 0,
3 3
a12
a13

1
1
3 3 r3 r1 = 0.
a13 a12
The vectors r1 and r3 are non-col1inear (and different from 0). Therefore
r3 r1 0
and
1
1
3 = 0,
3
a13 a12

hence,
a12 = a13 .
Similarly, one gets:
a12 = a23 (= a ).
We have re-derived the condition (11).
Taking into account that all the distances aij have the same value a, from the equation
(7) concerning the point P1, using the relation (2) we obtain:
G

mm
m1m2
(r2 r1 ) + G 1 3 3 (r3 r1 ) + m1 2 r1 = 0,
3
a
a
m
m
m
G 31 + G 32 G 33 m1r1 + m1 2 r1 = 0,
a
a
a
GM
= 2.
3
a

This is the condition (9). The same condition is got in result of similar calculations for the
points P2 and P3.

The method described here does not differ essentially from the first method. In fact
they are slight modifications of each other. However, it is interesting to notice how
application of a proper mathematical language, e.g. the vector product, simplifies the
calculations.
The relation (9) can be called a generalized Keplers law as, in fact, it is very similar
to the Keplers law but with respect to the many-body system. As far as I know this
generalized Keplers law was presented for the first time right at the 20th IPhO.
Marking scheme
1. the proof that = const
2. the conditions at the equilibrium (conditions for the forces
and their moments or extremum of the total potential energy)
3. the proof of the relation aij = a

1 point

4. the proof of the relation 2 a 3 = GM

2 points

3 points
4 points

Remarks and typical mistakes in the pupils' solutions


No type of error was observed as predominant in the pupils' solutions. Practically all the
mistakes can be put down to the students' scant experience in calculations and general lack of
skill. Several students misunderstood the text of the problem and attempted to prove that the
three masses should be equal. Of course, this was impossible. Moreover, it was pointless,
since the masses were given. Almost all the participants tried to solve the problem by
analyzing equilibrium of forces and/or their moments. Only one student tried to solve the
problem by looking for a minimum of the total potential energy (unfortunately, his solution
was not fully correct). Several participants solved the problem using a convenient reference
system: one mass in the origin and one mass on the x-axis. One of them received a special
prize.
Problem 3
The problem concerns investigation of transforming the electron microscope with
magnetic guiding of the electron beam (which is accelerated with the potential difference U =
511 kV) into a proton microscope (in which the proton beam is accelerated with the potential
difference U). For this purpose, solve the following two problems:
A. An electron after leaving a device, which accelerated it with the potential difference
U, falls into a region with an inhomogeneous field B generated with a system of stationary
coils L1, L2, , Ln. The known currents in the coils are i1, i2, , in, respectively.
What should the currents i1, i2, , in in the coils L1, L2, , Ln be, in order to guide
the proton (initially accelerated with the potential difference U) along the same trajectory
(and in the same direction) as that of the electron?
HINT: The problem can be solved by finding a condition under which the equation
describing the trajectory is the same in both cases. It may be helpful to use the relation:
p

d
1 d 2 1 d 2
p=
p =
p.
dt
2 dt
2 dt

B. How many times would the resolving power of the above microscope increase or
decrease if the electron beam were replaced with the proton beam? Assume that the resolving
power of the microscope (i.e. the smallest distance between two point objects whose circular
images can be just separated) depends only on the wave properties of the particles.
Assume that the velocities of the electrons and protons before their acceleration are zero,
and that there is no interaction between own magnetic moment of either electrons or protons
and the magnetic field. Assume also that the electromagnetic radiation emitted by the moving
particles can be neglected.
NOTE: Very often physicists use 1 electron-volt (1 eV), and its derivatives such as 1
keV or 1 MeV, as a unit of energy. 1 electron-volt is the energy gained by the electron that
passed the potential difference equal to 1 V.
Perform the calculations assuming the following data:
Rest energy of electron:
Rest energy of proton:

Ee = mec2 = 511 keV


Ep = mpc2 = 938 MeV

Solution
PART A

At the beginning one should notice that the kinetic energy of the electron accelerated
with the potential difference U = 511 kV equals to its rest energy E0 . Therefore, at least in the
case of the electron, the laws of the classical physics cannot be applied. It is necessary to use
relativistic laws.
The relativistic equation of motion of a particle with the charge e in the magnetic field
B has the following form:
d
p = FL
dt

where p = m0v denotes the momentum of the particle (vector) and

FL = ev B
is the Lorentz force (its value is evB and its direction is determined with the right hand rule).
m0 denotes the (rest) mass of the particle and v denotes the velocity of the particle. The
quantity is given by the formula:

1
1

v2
c2

The Lorentz force FL is perpendicular to the velocity v of the particle and to its momentum
p = m0v . Hence,

FL v = FL p = 0 .
Multiplying the equation of motion by p and making use of the hint given in the text of the
problem, we get:
1 d 2
p = 0.
2 dt

It means that the value of the particle momentum (and the value of the velocity) is constant
during the motion:
p = m0v = const;

v = const.

The same result can be obtained without any formulae in the following way:
The Lorentz force FL is perpendicular to the velocity v (and to the momentum p as
p = m0v ) and, as a consequence, to the trajectory of the particle. Therefore, there is no force
that could change the component of the momentum tangent to the trajectory. Thus, this
component, whose value is equal to the length of p , should be constant: p = const. (The same
refers to the component of the velocity tangent to the trajectory as p = m0v ).
Let s denotes the path passed by the particle along the trajectory. From the definition of
the velocity, we have:
ds
= v.
dt
Using this formula, we can rewrite the equation of motion as follows:
v

d
ds d
d
p=
p = p = FL ,
ds
dt ds
dt
d
F
p= L .
ds
v

Dividing this equation by p and making use of the fact that p = const, we obtain:
v

d p FL
=
ds p vp

and hence
d
F
t= L
ds
vp
where t = p / p = v / v is the versor (unit vector) tangent to the trajectory. The above equation
is exactly the same for both electrons and protons if and only if the vector quantity:
FL
vp
is the same in both cases.
Denoting corresponding quantities for protons with the same symbols as for the
electrons, but with primes, one gets that the condition, under which both electrons and protons
can move along the same trajectory, is equivalent to the equality:
FL F'L
.
=
vp v' p '
However, the Lorentz force is proportional to the value of the velocity of the particle,
and the directions of any two vectors of the following three: t (or v), FL, B determine the
direction of the third of them (right hand rule). Therefore, the above condition can be written
in the following form:
eB e' B'
.
=
p
p'

Hence,
B' =

e p'
p'
B = B.
e' p
p

This means that at any point the direction of the field B should be conserved, its
orientation should be changed into the opposite one, and the value of the field should be
multiplied by the same factor p'/p. The magnetic field B is a vector sum of the magnetic fields
of the coils that are arbitrarily distributed in the space. Therefore, each of this fields should be
scaled with the same factor -p'/p. However, the magnetic field of any coil is proportional to
the current flowing in it. This means that the required scaling of the fields can only be
achieved by the scaling of all the currents with the same factor -p'/p:
i 'n =

p'
in .
p

Now we shall determine the ratio p'/p. The kinetic energies of the particles in both cases
are the same; they are equal to Ek = e U = 511 keV. The general relativistic relation between
the total energy E of the particle with the rest energy E0 and its momentum p has the
following form:
E 2 = E02 + p 2c 2

where c denotes the velocity of light.


The total energy of considered particles is equal to the sum of their rest and kinetic
energies:
E = E0 + Ek .
Using these formulae and knowing that in our case Ek = e U = Ee , we determine the
momenta of the electrons (p) and the protons (p). We get:
a) electrons:
( Ee + Ee ) 2 = Ee2 + p 2c 2 ,

p=

Ee
3.
c

b) protons

( E p + Ee ) 2 = E p2 + p'2 c 2 ,
E
p' = e
c

Ep
E

+ 1 p .
Ee
Ee

Hence,
2

p' 1
=
p
3

E
Ep

+ 1 p 35.0

E
Ee

and
i 'n = 35.0 in .

It is worthwhile to notice that our protons are 'almost classical', because their kinetic
energy Ek (= Ee ) is small compared to the proton rest energy E p . Thus, one can expect that
the momentum of the proton can be determined, with a good accuracy, from the classical
considerations. We have:
Ee = Ek =

p '2
p '2 c 2
p '2 c 2
=
=
,
2m p 2m p c 2
2E p

p' =

1
2 Ee E p .
c

On the other hand, the momentum of the proton determined from the relativistic
formulae can be written in a simpler form since Ep/Ee 1. We get:

E
p' = e
c

E
E
Ep
E
E
E
1

+ 1 p = e 2 p + 1 e 2 p =
2 Ee E p .
c
Ee
c
Ee c
Ee
Ee

In accordance with our expectations, we have obtained the same result as above.
PART B

The resolving power of the microscope (in the meaning mentioned in the text of the
problem) is proportional to the wavelength, in our case to the length of the de Broglie wave:

h
p

where h denotes the Planck constant and p is the momentum of the particle. We see that is
inversely proportional to the momentum of the particle. Therefore, after replacing the electron
beam with the proton beam the resolving power will be changed by the factor p/p' 1/35. It
means that our proton microscope would allow observation of the objects about 35 times
smaller than the electron microscope.
Marking scheme
1. the relativistic equation of motion
2. independence of p and v of the time
3. identity of eB/p in both cases
4. scaling of the fields and the currents with the same factor
5. determination of the momenta (relativistically)
6. the ratio of the momenta (numerically)
7. proportionality of the resolving power to
8. inverse proportionality of to p
9. scaling of the resolving power

1 point
1 point
2 points
1 point
1 point
1 point
1 point
1 point
1 point

Remarks and typical mistakes in the pupils' solutions


Some of the participants tried to solve the problem by using laws of classical mechanics
only. Of course, this approach was entirely wrong. Some students tried to find the required
condition by equating "accelerations" of particles in both cases. They understood the
"acceleration" of the particle as a ratio of the force acting on the particle to the "relativistic"
mass of the particle. This approach is incorrect. First, in relativistic physics the relationship
between force and acceleration is more complicated. It deals with not one "relativistic" mass,

but with two "relativistic" masses: transverse and longitudinal. Secondly, identity of
trajectories need not require equality of accelerations.
The actual condition, i.e. the identity of eB/p in both cases, can be obtained from the
following two requirements:
1 in any given point of the trajectory the curvature should be the same in both cases;
2 in the vicinity of any given point the plane containing a small arc of the trajectory
should be oriented in space in both cases in the same way.
Most of the students followed the approach described just above. Unfortunately, many
forgot about the second requirement (they neglected the vector character of the quantity eB/p).
EXPERIMENTAL PROBLEM 1
The following equipment is provided:
1. Two piezoelectric discs of thickness 10 mm with evaporated electrodes (Fig. 4.1) fixed in
holders on the jaws of the calipers;
10 mm

Fig. 4.1

Electrodes

2. The calibrated sine wave oscillator with a photograph of the control panel, explaining the
functions of the switches and regulators;
3. A double channel oscilloscope with a photograph of the control panel, explaining the
functions of the switches and regulators;
4. Two closed plastic bags containing liquids;
5. A beaker with glycerin (for wetting the discs surfaces to allow better mechanical coupling);
6. Cables and a three way connector;
7. A stand for support the bags with the liquids;
8. Support and calipers.
A piezoelectric material changes its linear dimensions under the influence of an electric
field and vice-versa, the distortion of a piezoelectric material induces an electrical field.
Therefore, it is possible to excite the mechanical vibrations in a piezoelectric material by
applying an alternating electric field, and also to induce an alternating electric field by
mechanical vibrations.

The Organizing Committee planned to give another experimental problem: a problem on high Tc
superconductivity. Unfortunately, the samples of superconductors, prepared that time by a factory, were of very
poor quality. Moreover, they were provided after a long delay. Because of that the organizers decided to use this
problem, which was also prepared, but considered as a second choice.

A. Knowing that the velocity of longitudinal ultrasonic waves in the material of the disc
is about 4 10 3 m/s, estimate roughly the resonant frequency of the mechanical vibrations
parallel to the disc axis. Assume that the disc holders do no restrict the vibrations. (Note that
other types of resonant vibrations with lower or higher frequencies may occur in the discs.)
Using your estimation, determine experimentally the frequency for which the
piezoelectric discs work best as a transmitter-receiver set for ultrasound in the liquid. Wetting
surfaces of the discs before putting them against the bags improves penetration of the liquid in
the bag by ultrasound.
B. Determine the velocity of ultrasound for both liquids without opening the bags and
estimate the error.
C. Determine the ratio of the ultrasound velocities for both liquids and its error.
Complete carefully the synopsis sheet. Your report should, apart from the synopsis
sheet, contain the descriptions of:
- method of resonant frequency estimation;
- methods of measurements;
- methods of estimating errors of the measured quantities and of final results.
Remember to define all the used quantities and to explain the symbols.

Synopsis Sheet1
Formula for estimating the resonant frequency:

Results (with units):

A
Measured best transmitter frequency (with units):

Definition of measured quantity:

Symbol:

Error:

Results:

Final formula for ultrasound velocity in liquid:


Velocity of ultrasound (with units):

Error:

Liquid A
Liquid B
Ratio of velocities:

Solution (draft) 1

Error:

In the real Synopsis Sheet the students had more space for filling.

Error:

A. As the holders do not affect vibrations of the disc we may expect antinodes on the
flat surfaces of the discs (Fig. 4.2; geometric proportions not conserved). One of the
frequencies is expected for
l = 12 =

v
,
2f

where v denotes the velocity of longitudinal ultrasonic wave (its value is given in the text of
the problem), f - the frequency and l - the thickness of the disc. Thus:
f =

v
.
2l

Numerically f = 2 10 5 Hz = 200 kHz.


Mode considered in the problem
(schematically)

Axis of
the disc

l = /2
Fig. 4.2

One should stress out that different modes of vibrations can be excited in the disc with
height comparable to its diameter. We confine our considerations to the modes related to
longitudinal waves moving along the axis of the disc as the sound waves in liquids are
longitudinal. We neglect coupling between different modes and require antinodes exactly at
the flat parts of the disc. We assume also that the piezoelectric effect does not affect velocity
of ultrasound. For these reasons the frequency just determined should be treated as only a
rough approximation. However, it indicates that one should look for the resonance in vicinity
of 200 kHz.
The experimental set-up is shown in Fig. 4.3. The oscillator (generator) is connected to
one of the discs that works as a transmitter and to one channel of the oscilloscope. The second
disc is connected to the second channel of the oscilloscope and works as a receiver. Both discs
are placed against one of the bags with liquid (Fig. 4.4). The distance d can be varied.

This draft solution is based on the camera-ready text of the more detailed solution prepared by Dr. Andrzej
Kotlicki and published in the proceedings [3]

Y1
Oscillator
(Generator)

Oscilloscope

Y2

Fig. 4.3

Bag with liquid

Fig. 4.4

One searches for the resonance by slowly changing the frequency of the oscillator in the
range 100 1000 kHz and watching the signal on the oscilloscope. In this way the students
could find a strong resonance at frequency f 220 kHz. Other resonance peaks could be
found at about 110 kHz and 670 kHz. They should have been neglected as they are
substantially weaker. (They correspond to some other modes of vibrations.) Accuracy of these
measurements was 10 kHz (due to the width of the resonance and the accuracy of the scale on
the generator).
B. The ultrasonic waves pass through the liquid and generate an electric signal in the
receiver. Using the same set-up (Fig. 4.3 and 4.4) we can measure dependence of the phase
shift between the signals at Y1 and Y2 vs. distance between the piezoelectric discs d at the
constant frequency found in point A. This phase shift is = 2df / vl + 0 , where vl denotes
velocity of ultrasound in the liquid. 0 denotes a constant phase shift occurring when
ultrasound passes trough the bag walls (possibly zero). The graph representing dependence
d ( ) should be a straight line. Its slope allows to determine vl and its error. In general, the

measurements of are difficult for many reflections in the bag, which perturb the signal.
One of the best ways is to measure d only for = n (n - integer) as such points can be
found rather easy. Many technical details concerning measurements can be found in [3] (pp.
37 and 38).
The liquids given to the students were water and glycerin. In the standard solution the
author of the problem received the following values:
vwater = (1.50 0.10) 10 3 m/s; vglycerin = (1.96 0.10) 10 3 m/s.
The ratio of these values was 1.31 0.15 .
The ultrasonic waves are partly reflected or scattered by the walls of the bag. This effect
somewhat affects measurements of the phase shift. To minimize its role one can measure the
phase shift (for a given distance) or distance (at the same phase shift) several times, each time
changing the shape of the bag. As regards errors in determination of velocities it is worth to
mention that the most important factor affecting them was the error in determination of the
frequency. This error, however, practically does not affect the ratio of velocities.
Marking Scheme
Frequency estimation
1.
2.
3.
4.
5.

Formula
Result (with units)
Method of experimental determining the resonance frequency
Result (if within 5% of standard value)
Error

1 point
1 point
1 point
2 points
1 point

Measurements of velocities
1.
2.
3.
4.
5.
6.

Explanation of the method


2 points
Proper number of measurements in each series
3 points
Result for velocity in the first liquid (if within 5% of standard value) 2 points
Error of the above
1 point
Result for velocity in the second liquid (if within 5% of standard value) 2 points
Error of the above
1 point
Ratio of velocities

1. Result (if within 3% of standard value)


2. Error of the above

2 points
1 point

Typical mistakes
The results of this problem were very good (more than a half of competitors obtained
more than 15 points). Nevertheless, many students encountered some difficulties in estimation
of the frequency. Some of them assumed presence of nodes at the flat surfaces of the discs
(this assumption is not adequate to the situation, but accidentally gives proper formula). In
part B some students tried to find distances between nodes and antinodes for ultrasonic
standing wave in the liquid. This approach gave false results as the pattern of standing waves
in the bag is very complicated and changes when the shape of the bag is changed.
Acknowledgement

I would like to thank very warmly to Prof. Jan Mostowski and Dr. Andrzej Wysmoek
for reading the text of this article and for valuable critical remarks. I express special thanks to
Dr. Andrzej Kotlicki for critical reviewing the experimental part of the article and for a
number of very important improvements.
Literature
[1] Waldemar Gorzkowski and Andrzej Kotlicki, XX Midzynarodowa Olimpiada
Fizyczna - cz. I, Fizyka w Szkole nr 1/90, pp. 34 - 39
[2] Waldemar Gorzkowski, XX Midzynarodowa Olimpiada Fizyczna - cz. II, Fizyka
w Szkole nr 2/3-90, pp. 23 - 32
[3] XX International Physics Olympiad - Proceedings of the XX International Physics
Olympiad, Warsaw (Poland), July 16 - 24, 1989, ed. by W. Gorzkowski, World Scientific
Publishing Company, Singapore 1990 [ISBN 981-02-0084-6]

7+(VW,17(51$7,21$/3+<6,&62/<03,$'
*521,1*(17+(1(7+(5/$1'6
+DQV-RUGHQV
'HSDUWPHQWRI6FLHQFHDQG7HFKQRORJ\8QLYHUVLW\RI*URQLQJHQ7KH1HWKHUODQGV
4XHVWLRQ

;UD\'LIIUDFWLRQIURPDFU\VWDO

:HZLVKWRVWXG\;UD\GLIIUDFWLRQE\DFXELFFU\VWDOODWWLFH7RGRWKLVZHVWDUWZLWKWKH
GLIIUDFWLRQRIDSODQHPRQRFKURPDWLFZDYHWKDWIDOOVSHUSHQGLFXODUO\RQDGLPHQVLRQDO
JULGWKDWFRQVLVWVRI1[1VOLWVZLWKVHSDUDWLRQVGDQGG7KHGLIIUDFWLRQSDWWHUQLV
REVHUYHGRQDVFUHHQDWDGLVWDQFH/IURPWKHJULG7KHVFUHHQLVSDUDOOHOWRWKHJULGDQG
/LVPXFKODUJHUWKDQGDQGG
D

'HWHUPLQHWKHSRVLWLRQVDQGZLGWKVRIWKHSULQFLSDOPD[LPXPRQWKHVFUHHQ
7KHZLGWKLVGHILQHGDVWKHGLVWDQFHEHWZHHQWKHPLQLPDRQHLWKHUVLGHRIWKH
PD[LPD

:HFRQVLGHUQRZDFXELFFU\VWDOZLWKODWWLFHVSDFLQJDDQGVL]H1D[1D[1D1LV
PXFKVPDOOHUWKDQ17KHFU\VWDOLVSODFHGLQDSDUDOOHO;UD\EHDPDORQJWKH]D[LVDW
DQDQJOH VHH)LJ 7KHGLIIUDFWLRQSDWWHUQLVDJDLQREVHUYHGRQDVFUHHQDWDJUHDW
GLVWDQFHIURPWKHFU\VWDO

Figure 1 Diffraction of a parallel X-ray beam along the z-axis.


The angle between the crystal and the y-axis is .
E


&DOFXODWHWKHSRVLWLRQDQGZLGWKRIWKHPD[LPDDVDIXQFWLRQRIWKHDQJOH IRU
VPDOO 
:KDWLQSDUWLFXODUDUHWKHFRQVHTXHQFHVRIWKHIDFWWKDW11

7KHGLIIUDFWLRQSDWWHUQFDQDOVREHGHULYHGE\PHDQVRI%UDJJ
VWKHRU\LQZKLFKLWLV
DVVXPHGWKDWWKH;UD\VDUHUHIOHFWHGIURPDWRPLFSODQHVLQWKHODWWLFH7KHGLIIUDFWLRQ
SDWWHUQWKHQDULVHVIURPLQWHUIHUHQFHRIWKHVHUHIOHFWHGUD\VZLWKHDFKRWKHU
F

6KRZWKDWWKLVVRFDOOHG%UDJJUHIOHFWLRQ\LHOGVWKHVDPHFRQGLWLRQVIRUWKHPD[LPD
DVWKRVHWKDW\RXIRXQGLQE

,QVRPHPHDVXUHPHQWVWKHVRFDOOHGSRZGHUPHWKRGLVHPSOR\HG$EHDPRI;UD\VLV
VFDWWHUHGE\DSRZGHURIYHU\PDQ\VPDOOFU\VWDOV 2IFRXUVHWKHVL]HVRIWKHFU\VWDOVDUH
PXFKODUJHUWKDQWKHODWWLFHVSDFLQJD 
6FDWWHULQJRI;UD\VRIZDYHOHQJWKQPE\3RWDVVLXP&KORULGH>.&O@ ZKLFKKDVD
FXELFODWWLFHVHH)LJ UHVXOWVLQWKHSURGXFWLRQRIFRQFHQWULFGDUNFLUFOHVRQD
SKRWRJUDSKLFSODWH7KHGLVWDQFHEHWZHHQWKHFU\VWDOVDQGWKHSODWHLVPDQGWKH
UDGLXVRIWKHVPDOOHVWFLUFOHLVP VHH)LJ .DQG&OLRQVKDYHDOPRVWWKHVDPH
VL]HDQGWKH\PD\EHWUHDWHGDVLGHQWLFDOVFDWWHULQJFHQWUHV

Figure 2. The cubic latice of


Potassium Chloride in which
the K+ and Cl-wons have
almost the same size.
G

Figure 3. Scattering of X-rays by a powder of KCl crystals


results in the production of concentric dark circles on a
photographic plate.

&DOFXODWHWKHGLVWDQFHEHWZHHQWZRQHLJKERXULQJ.LRQVLQWKHFU\VWDO

4XHVWLRQ

(OHFWULFH[SHULPHQWVLQWKHPDJQHWRVSKHUHRIWKHHDUWK

,Q0D\WKHVSDFHVKLS$WODQWLVZLOOEHSODFHGLQRUELWDURXQGWKHHDUWK:HVKDOO
DVVXPHWKDWWKLVRUELWZLOOEHFLUFXODUDQGWKDWLWOLHVLQWKHHDUWK
VHTXDWRULDOSODQH
$WVRPHSUHGHWHUPLQHGPRPHQWWKHVSDFHVKLSZLOOUHOHDVHDVDWHOOLWH6ZKLFKLVDWWDFKHG
WRDFRQGXFWLQJURGRIOHQJWK/:HVXSSRVHWKDWWKHURGLVULJLGKDVQHJOLJLEOHPDVV
DQGLVFRYHUHGE\DQHOHFWULFDOLQVXODWRU:HDOVRQHJOHFWDOOIULFWLRQ/HWEHWKHDQJOH
WKDWWKHURGPDNHVWRWKHOLQHEHWZHHQWKH$WODQWLVDQGWKHFHQWUHRIWKHHDUWK VHH)LJ
 
6DOVROLHVLQWKHHTXDWRULDOSODQH
$VVXPHWKDWWKHPDVVRIWKHVDWHOOLWH
LVPXFKVPDOOHUWKDQWKDWRIWKH
$WODQWLVDQGWKDW/LVPXFKVPDOOHU
WKDQWKHUDGLXVRIWKHRUELW

D 'HGXFHIRUZKLFKYDOXH V RI


WKHFRQILJXUDWLRQRIWKH
VSDFHVKLSDQGVDWHOOLWHUHPDLQ
XQFKDQJHG ZLWKUHVSHFWWRWKH
Figure 1 The spaceship Atlantis (A) with a satellite
HDUWK ",QRWKHUZRUGVIRU
(S)
in an orbit around the earth. The orbit lies in the
ZKLFKYDOXH V RILV
earths equatorial plane.
FRQVWDQW"
The magnetic field (B) is perpendicular to the
diagram and is directed towards the reader.

D 'LVFXVVWKHVWDELOLW\RIWKHHTXLOLEULXPIRUHDFKFDVH
6XSSRVHWKDWDWDJLYHQPRPHQW
WKHURGGHYLDWHVIURPWKHVWDEOHFRQILJXUDWLRQE\DVPDOODQJOH7KHV\VWHPZLOOEHJLQWR
VZLQJOLNHDSHQGXOXP
E

([SUHVVWKHSHULRGRIWKHVZLQJLQJLQWHUPVRIWKHSHULRGRIUHYROXWLRQRIWKHV\VWHP
DURXQGWKHHDUWK

,Q)LJWKHPDJQHWLFILHOGRIWKHHDUWKLVSHUSHQGLFXODUWRWKHGLDJUDPDQGLVGLUHFWHG
WRZDUGVWKHUHDGHU'XHWRWKHRUELWDOYHORFLW\RIWKHURGDSRWHQWLDOGLIIHUHQFHDULVHV
EHWZHHQLWVHQGV7KHHQYLURQPHQW WKHPDJQHWRVSKHUH LVDUDUHILHGLRQLVHGJDVZLWKD
YHU\JRRGHOHFWULFDOFRQGXFWLYLW\&RQWDFWZLWKWKHLRQLVHGJDVLVPDGHE\PHDQVRI
HOHFWURGHVLQ$ WKH$WODQWLV DQG6 WKHVDWHOOLWH $VDFRQVHTXHQFHRIWKHPRWLRQD
FXUUHQW,IORZVWKURXJKWKHURG
F ,QZKLFKGLUHFWLRQGRHVWKHFXUUHQWIORZWKURXJKWKHURG" 7DNH 
'DWD

WKHSHULRGRIWKHRUELW
OHQJWKRIWKHURG
PDJQHWLFILHOGVWUHQJWKRIWKHHDUWDWWKHKHLJKW
RIWKHVDWHOOLWH
WKHPDVVRIWKHVKXWWOH$WODQWLV

7 V
/ P
% :EP
P NJ

1H[WDFXUUHQWVRXUFHLQVLGHWKHVKXWWOHLVLQFOXGHGLQWKHFLUFXLWZKLFKPDLQWDLQHVDQHW
GLUHFWFXUUHQWRI$LQWKHRSSRVLWHGLUHFWLRQ
F +RZORQJPXVWWKLVFXUUHQWEHPDLQWDLQHGWRFKDQJHWKHDOWLWXGHRIWKHRUELWE\
P
$VVXPHWKDWUHPDLQV]HUR,JQRUHDOOFRQWULEXWLRQVIURPFXUUHQWVLQWKH
PDJQHWRVSKHUH
 'RHVWKHDOWLWXGHGHFUHDVHRULQFUHDVH"
4XHVWLRQ

7KHURWDWLQJQHXWURQVWDU

$
PLOOLVHFRQGSXOVDU
LVDVRXUFHRIUDGLDWLRQLQWKHXQLYHUVHWKDWHPLWVYHU\VKRUWSXOVHV
ZLWKDSHULRGRIRQHWRVHYHUDOPLOOLVHFRQGV7KLVUDGLDWLRQLVLQWKHUDGLRUDQJHRI
ZDYHOHQJWKVDQGDVXLWDEOHUDGLRUHFHLYHUFDQEHXVHGWRGHWHFWWKHVHSDUDWHSXOVHVDQG
WKHUHE\WRPHDVXUHWKHSHULRGZLWKJUHDWDFFXUDF\
7KHVHUDGLRSXOVHVRULJLQDWHIURPWKHVXUIDFHRIDSDUWLFXODUVRUWRIVWDUWKHVRFDOOHG
QHXWURQVWDU7KHVHVWDUVDUHYHU\FRPSDFWWKH\KDYHDPDVVRIWKHVDPHRUGHURI
PDJQLWXGHDVWKDWRIWKHVXQEXWWKHLUUDGLXVLVRQO\DIHZWHQVRINLORPHWHUV7KH\VSLQ
YHU\TXLFNO\%HFDXVHRIWKHIDVWURWDWLRQDQHXWURQVWDULVVOLJKWO\IODWWHQHG REODWH 
$VVXPHWKHD[LDOFURVVVHFWLRQRIWKHVXUIDFHWREHDQHOOLSVHZLWKDOPRVWHTXDOD[HV
/HWUSEHWKHSRODUDQGUHWKHHTXDWRULDOUDGLLDQGOHWXVGHILQHWKHIODWWHQLQJIDFWRUE\

&RQVLGHUDQHXWURQVWDUZLWK



D

DPDVVRI

DQDYHUDJHUDGLXVRI
DQG DURWDWLRQSHULRGRI

NJ
P
V

&DOFXODWHWKHIODWWHQLQJIDFWRUJLYHQWKDWWKHJUDYLWDWLRQDOFRQVWDQWLV
1PNJ

,QWKHORQJUXQ RYHUPDQ\\HDUV WKHURWDWLRQRIWKHVWDUVORZVGRZQGXHWRHQHUJ\ORVV


DQGWKLVOHDGVWRDGHFUHDVHLQWKHIODWWHQLQJ7KHVWDUKDVKRZHYHUDVROLGFUXVWWKDW
IORDWVRQDOLTXLGLQWHULRU7KHVROLGFUXVWUHVLVWVDFRQWLQXRXVDGMXVWPHQWWRHTXLOLEULXP
VKDSH,QVWHDGVWDUTXDNHVRFFXUZLWKVXGGHQFKDQJHVLQWKHVKDSHRIWKHFUXVWWRZDUGV
HTXLOLEULXP'XULQJDQGDIWHUVXFKDVWDUTXDNHWKHDQJXODUYHORFLW\LVREVHUYHGWR
FKDQJHDFFRUGLQJWRILJXUH

DQJXODUYHORFLW\
RIWKHFUXVW V

Figure 1

time (days) -->

A sudden change in the shape of the crust of a


neutron star results in a sudden change of the
angular velocity.
E

&DOFXODWHWKHDYHUDJHUDGLXVRIWKHOLTXLGLQWHULRUXVLQJWKHGDWDRI)LJ0DNH
WKHDSSUR[LPDWLRQWKDWWKHGHQVLWLHVRIWKHFUXVWDQGWKHLQWHULRUDUHWKHVDPH
,JQRUHWKHFKDQJHLQVKDSHRIWKHLQWHULRU 

4XHVWLRQ

'HWHUPLQDWLRQRIWKHHIILFLHQF\RID/('

,QWURGXFWLRQ
,QWKLVH[SHULPHQWZHVKDOOXVHWZRPRGHUQVHPLFRQGXFWRUVWKHOLJKWHPLWWLQJGLRGH
/(' DQGWKHSKRWRGLRGH 3' ,QD/('SDUWRIWKHHOHFWULFDOHQHUJ\LVXVHGWRH[FLWH
HOHFWURQVWRKLJKHUHQHUJ\OHYHOV:KHQVXFKDQH[FLWHGHOHFWURQIDOOVEDFNWRDORZHU
HQHUJ\OHYHODSKRWRQZLWKHQHUJ\(SKRWRQLVHPLWWHGZKHUH

+HUHKLV3ODQFN
VFRQVWDQWFLVWKHVSHHGRIOLJKWDQG LVWKHZDYHOHQJWKRIWKHHPLWWHG
OLJKW7KHHIILFLHQF\RIWKH/('LVGHILQHGWREHWKHUDWLREHWZHHQWKHUDGLDWHGSRZHU
DQGWKHHOHFWULFDOSRZHUXVHG3/('

,QDSKRWRGLRGHUDGLDQWHQHUJ\LVWUDQVIRUPHGLQWRHOHFWULFDOHQHUJ\:KHQOLJKWIDOOVRQ
WKHVHQVLWLYHVXUIDFHRIDSKRWRGLRGHVRPH EXWQRWDOO RIWKHSKRWRQVIUHHVRPH EXW
QRWDOO RIWKHHOHFWURQVIURPWKHFU\VWDOVWUXFWXUH7KHUDWLREHWZHHQWKHQXPEHURI
LQFRPLQJSKRWRQVSHUVHFRQG1SDQGWKHQXPEHURIIUHHGHOHFWURQVSHUVHFRQG1HLV
FDOOHGWKHTXDQWXPHIILFLHQF\TS

7KHH[SHULPHQW
7KHSXUSRVHRIWKLVH[SHULPHQWLVWRGHWHUPLQHWKHHIILFLHQF\RID/('DVDIXQFWLRQRI
WKHFXUUHQWWKDWIORZVWKURXJKWKH/('7RGRWKLVZHZLOOPHDVXUHWKHLQWHQVLW\RIWKH
HPLWWHGOLJKWZLWKDSKRWRGLRGH7KH/('DQGWKH3'KDYHEHHQPRXQWHGLQWZR
ER[HVDQGWKH\DUHFRQQHFWHGWRDFLUFXLWSDQHO )LJ %\PHDVXULQJWKHSRWHQWLDO
GLIIHUHQFHDFURVVWKH/('DQGDFURVVWKHUHVLVWRUV5DQG5RQHFDQGHWHUPLQHERWK
WKHSRWHQWLDOGLIIHUHQFHVDFURVVDQGWKHFXUUHQWVIORZLQJWKURXJKWKH/('DQGWKH3'
:HXVHWKHPXOWLPHWHUWRPHDVXUH92/7$*(6RQO\7KLVLVGRQHE\WXUQLQJWKHNQRE
WRSRVLWLRQ
9
7KHPHWHUVHOHFWVWKHDSSURSULDWHVHQVLWLYLW\UDQJHDXWRPDWLFDOO\,IWKH
GLVSOD\LVQRWRQ$872VZLWFKRIIDQGSXVKRQ9DJDLQ&RQQHFWLRQ&20DQG
96
7KHER[FRQWDLQLQJWKHSKRWRGLRGHDQGWKHER[FRQWDLQLQJWKH/('FDQEHPRYHG
IUHHO\RYHUWKHERDUG,IERWKER[HVDUHSRVLWLRQHGRSSRVLWHWRHDFKRWKHUWKHQWKH/('
WKH3'DQGWKHKROHLQWKHER[FRQWDLQLQJWKH3'UHPDLQLQDVWUDLJKWOLQH
'DWD7KHTXDQWXPHIILFLHQF\RIWKHSKRWRGLRGH
7KHGHWHFWLRQVXUIDFHRIWKH3'LV
7KHZDYHOHQJWKRIWKHOLJKWHPLWWHGIURPWKH/('LV
7KHLQWHUQDOUHVLVWDQFHRIWKHYROWPHWHULV

TS 
[PP
QP
06LQWKHUDQJHXSWR
P9
06LQWKHRWKHUUDQJHV
7KHUDQJHLVLQGLFDWHGE\VPDOOQXPEHUVRQWKHGLVSOD\
3ODQFN
VFRQVWDQW
K -V
7KHHOHPHQWDU\TXDQWXPRIFKDUJH
H &
7KHVSHHGRIOLJKWLQYDFXR
F PV

Figure 1.
5 6
5 YDULDEOHUHVLVWRU
5 06
7KHSRLQWVODEHOOHGDQG
DUHPHDVXULQJSRLQWV
Figure 2 The experimental setup: a board and the
two boxes containing the LED and the photo-diode.

,QVWUXFWLRQV


%HIRUHZHFDQGHWHUPLQHWKHHIILFLHQF\RIWKH/('ZHPXVWILUVWFDOLEUDWHWKH
SKRWRGLRGH7KHSUREOHPLVWKDWZHNQRZQRWKLQJDERXWWKH/('
6KRZH[SHULPHQWDOO\WKDWWKHUHODWLRQEHWZHHQWKHFXUUHQWIORZLQJWKURXJKWKH
SKRWRGLRGHDQGWKHLQWHQVLW\RIOLJKWIDOOLQJRQLW,>-VP@LVOLQHDU



'HWHUPLQHWKHFXUUHQWIRUZKLFKWKH/('KDVPD[LPDOHIILFLHQF\



&DUU\RXWDQH[SHULPHQWWRPHDVXUHWKHPD[LPDO DEVROXWH HIILFLHQF\RIWKH/('

1RPDUNV SRLQWV ZLOOEHDOORFDWHGIRUDQHUURUDQDO\VLV LQ7+,6H[SHULPHQWRQO\ 


3OHDVHVXPPDUL]HGDWDLQWDEOHVDQGJUDSKVZLWKFOHDULQGLFDWLRQVRITXDQWLWLHV DQG
XQLWV 

4XHVWLRQ

'HWHUPLQDWLRQRIWKHUDWLRRIWKHPDJQHWLFILHOGVWUHQJWKVRI
WZRGLIIHUHQWPDJQHWV

,QWURGXFWLRQ
:KHQDFRQGXFWRUPRYHVLQDPDJQHWLFILHOGFXUUHQWVDUHLQGXFHGWKHVHDUHWKH
VRFDOOHGHGG\FXUUHQWV$VDFRQVHTXHQFHRIWKHLQWHUDFWLRQEHWZHHQWKHPDJQHWLFILHOG
DQGWKHLQGXFHGFXUUHQWVWKHPRYLQJFRQGXFWRUVXIIHUVDUHDFWLYHIRUFH7KXVDQ
DOXPLQLXPGLVNWKDWURWDWHVLQWKHQHLJKERXUKRRGRIDVWDWLRQDU\PDJQHWH[SHULHQFHVD
EUDNLQJIRUFH
0DWHULDODYDLODEOH









$VWDQG
$FODPS
$QKRPRJHQLXVDOXPLQLXPGLVNRQDQD[OHLQDKROGHUWKDWFDQURWDWH
7ZRPDJQHWV7KHJHRPHWU\RIHDFKLVWKHVDPH XSWR HDFKFRQVLVWVRIDFOLS
FRQWDLQLQJWZRVPDOOPDJQHWVRILGHQWLFDOPDJQHWL]DWLRQDQGDUHDWKHZKROH
SURGXFLQJDKRPRJHQLXVILHOG%RU%
7ZRZHLJKWV2QHZHLJKWKDVWZLFHWKHPDVV XSWR RIWKHRWKHU
$VWRSZDWFK
$UXOHU

Figure 1.
7KHH[SHULPHQW
7KHDOXPLQLXPGLVNLVIL[HGWRDQD[OHDURXQGZKLFKDFRUGLVZUDSSHG$ZHLJKWKDQJV
IURPWKHFRUGDQGZKHQWKHZHLJKWLVUHOHDVHGWKHGLVNDFFHOHUDWHVXQWLODFRQVWDQW
DQJXODUYHORFLW\LVUHDFKHG7KHWHUPLQDOVSHHGGHSHQGVDPRQJRWKHUWKLQJVRQWKH
PDJQLWXGHRIWKHPDJQHWLFILHOGVWUHQJWKRIWKHPDJQHW
7ZRPDJQHWVRIGLIIHUHQWILHOGVWUHQJWKV%RU%DUHDYDLODEOH(LWKHUFDQEHILWWHGRQWR
WKHKROGHUWKDWFDUULHVWKHDOXPLQLXPGLVNWKH\PD\EHLQWHUFKDQJHG
,QVWUXFWLRQV


7KLQNRIDQH[SHULPHQWLQZKLFKWKHUDWLRRIWKHPDJQHWLFILHOGVWUHQJWKV%DQG%
RIWKHWZRPDJQHWVFDQEHPHDVXUHGDVDFFXUDWHO\DVSRVVLEOH



*LYHDVKRUWWKHRUHWLFDOWUHDWPHQWLQGLFDWLQJKRZRQHFDQREWDLQWKHUDWLRIURP
WKHPHDVXUHPHQWV



&DUU\RXWWKHH[SHULPHQWDQGGHWHUPLQHWKHUDWLR



*,9($1(5525(67,0$7,21

8VHRIWKHVWRSZDWFK

Figure 2.
7KHVWRSZDWFKKDVWKUHHEXWWRQV66DQG6 VHH)LJ 
%XWWRQ6WRJJOHVEHWZHHQWKHGDWHWLPHDQGWKHVWRSZDWFKPRGHV6ZLWFKWRWKH
VWRSZDWFKPRGH2QHVKRXOGVHHWKLV

2QSUHVVLQJ6RQFHWKHVWRSZDWFKEHJLQVWLPLQJ7RVWRSLWSUHVV6DVHFRQGWLPH
7KHVWRSZDWFKFDQEHUHVHWWR]HURE\SUHVVLQJ6RQFH

6ROXWLRQRITXHVWLRQ
D

&RQVLGHUILUVWWKH[GLUHFWLRQ,IZDYHVFRPLQJIURPQHLJKERXULQJVOLWV ZLWK
VHSDUDWLRQG WUDYHUVHSDWKVRIOHQJWKVWKDWGLIIHUE\
ZKHUHQLVDQLQWHJHUWKHQDSULQFLSDOPD[LPXPRFFXUV7KHSRVLWLRQRQWKH
VFUHHQ LQWKH[GLUHFWLRQ LV

VLQFHGG
7KHSDWKGLIIHUHQFHEHWZHHQWKHPLGGOHVOLWDQGRQHRIWKHVOLWVDWWKHHGJHLVWKHQ

,IRQWKHRWKHUKDQGWKLVSDWKGLIIHUHQFHLV

WKHQWKHILUVWPLQLPXPQH[WWRWKHSULQFLSDOPD[LPXPRFFXUV7KHSRVLWLRQRIWKLV
PLQLPXPRQWKHVFUHHQLVJLYHQE\

7KHZLGWKRIWKHSULQFLSDOPD[LPXPLVDFFRUGLQJO\

$VLPLODUWUHDWPHQWFDQEHPDGHIRUWKH\GLUHFWLRQLQZKLFKWKHUHDUH1VOLWVZLWK
VHSDUDWLRQG7KHSRVLWLRQVDQGZLGWKVRIWKHSULQFLSDOPD[LPDODUH

$QDOWHUQDWLYHPHWKRGRIVROXWLRQLVWRFDOFXODWHWKHLQWHQVLW\IRUWKHGLPHQVLRQDO
JULGDVDIXQFWLRQRIWKHDQJOHWKDWWKHEHDPPDNHVZLWKWKHVFUHHQ
E

,QWKH[GLUHFWLRQWKHEHDP
VHHV
DJULGZLWKVSDFLQJDVRWKDWLQWKLVGLUHFWLRQZH
KDYH

,QWKH\GLUHFWLRQWKHEHDP
VHHV
DJULGZLWKHIIHFWLYHVSDFLQJDFRV  
$QDORJRXVO\ZHREWDLQ

,QWKH]GLUHFWLRQWKHEHDP
VHHV
DJULGZLWKHIIHFWLYHVSDFLQJDVLQ  7KLVJLYHV
ULVHWRSULQFLSDOPD[LPDZLWKSRVLWLRQDQGZLGWK

7KLVSDWWHUQLVVXSHULPSRVHGRQWKHSUHYLRXVRQH6LQFHVLQ  LVYHU\VPDOORQO\
WKH]HURWKRUGHUSDWWHUQZLOOEHVHHQDQGLWLVYHU\EURDGVLQFH1VLQ  1
7KHGLIIUDFWLRQSDWWHUQIURPDSODQHZDYHIDOOLQJRQDWKLQSODWHRIDFXELFFU\VWDO
DWDVPDOODQJOHRILQFLGHQFHWRWKHQRUPDOZLOOEHDOPRVWLGHQWLFDOWRWKDWIURPD
WZRGLPHQVLRQDOJULG
F

,Q%UDJJUHIOHFWLRQWKHSDWKGLIIHUHQFHIRUFRQVWUXFWLYHLQWHUIHUHQFHEHWZHHQ
QHLJKERXULQJSODQHV

+HUH LVWKHDQJOHRIGLIIUDFWLRQ
7KLVLVWKHVDPHFRQGLWLRQIRUDPD[LPXPDVLQVHFWLRQE
G

)RUWKHGLVWDQFHDEHWZHHQQHLJKERXULQJ.LRQVZHKDYH

0DUNLQJ%UHDNGRZQ
D
E
F
G

SRVLWLRQRISULQFLSDOPD[LPD
ZLGWKRISULQFLSDOPD[LPD
ODWWLFHFRQVWDQWV
HIIHFWRIWKLFNQHVV






%UDJJUHIOHFWLRQ
&DOFXODWLRQRI..VSDFLQJ




6ROXWLRQRITXHVWLRQ
D 6LQFHPPWKH$WODQWLVWUDYHOVDURXQGWKHHDUWKZLWKDFRQVWDQWVSHHG7KH
PRWLRQRIWKHVDWHOOLWHLVFRPSRVHGRIWKHFLUFXODUPRWLRQRIWKH$WODQWLVDERXWWKH
HDUWKDQG SRVVLEO\ DFLUFXODUPRWLRQRIWKHVDWHOOLWHDERXWWKH$WODQWLV
)RUPZHKDYH

)RUPZHKDYH

8VLQJWKHDSSUR[LPDWLRQ

DQGHTXDWLRQ  RQHILQGV

VR
,ILVFRQVWDQW  


!VLQ   
!FRV   

!
!

   %
 %  %

D 7KHVLWXDWLRQLVVWDEOHLIWKHPRPHQW

PDQQHURSSRVHGWRWKDWLQZKLFKWKHVLJQRIFKDQJHV
VLJQ 









FKDQJHVVLJQLQD




%
%
%
%


VLJQ 0  
 








%
%
%
%

7KHHTXLOLEULXPDERXWWKHDQJOHVHQ%LVWKXVVWDEOHZKHUHDVWKDWDURXQG%
DQG%LVXQVWDEOH
E

)RUVPDOOYDOXHVRIHTXDWLRQ  EHFRPHV

7KLVLVWKHHTXDWLRQRIDVLPSOHKDUPRQLFPRWLRQ
7KHVTXDUHRIWKHDQJXODUIUHTXHQF\LV
VR

F $FFRUGLQJWR/HQ]
VODZWKHUHZLOOEHDFXUUHQWIURPWKHVDWHOOLWH 6 WRZDUGVWKH
VKXWWOH $ 
F )RUWKHWRWDOHQHUJ\RIWKHV\VWHPZHKDYH

$VPDOOFKDQJHLQWKHUDGLXVRIWKHRUELWFRUUHVSRQGVWRDFKDQJHLQWKHHQHUJ\RI

,QWKHVLWXDWLRQXQGHUFHQHUJ\LVDEVRUEHGIURPWKHV\VWHPDVDFRQVHTXHQFHRI
ZKLFKWKHUDGLXVRIWKHRUELWZLOOGHFUHDVH
,VDFXUUHQWVRXUFHLQVLGHWKHVKXWWOHLQFOXGHGLQWKHFLUFXLWZKLFKPDLQWDLQVDQHW
FXUUHQWLQWKHRSSRVLWHGLUHFWLRQHQHUJ\LVDEVRUEHGE\WKHV\VWHPDVD
FRQVHTXHQFHRIZKLFKWKHUDGLXVRIWKHRUELWZLOOLQFUHDVH
)URPWKHDVVXPSWLRQVLQFZHKDYH

1XPHULFDODSSOLFDWLRQJLYHVIRUWKHWLPHWVZKLFKLVDERXWWKHSHULRG
RIWKHV\VWHP

0DUNLQJEUHDNGRZQ
D



D



E





$WODQWLVLQXQLIRUPFLUFXODUPRWLRQ
FDOFXODWLRQRIWKHSHULRG6
HTXDWLRQRIPRWLRQRIWKHVDWHOOLWH
HTXDWLRQRIPRWLRQIRUVPDOODQJOHV
SHULRGRIRVFLOODWLRQV







F



F FDOFXODWLRQRIWKHWLPHWKHFXUUHQWKDVWREHPDLQWDLQHG
 LQFUHDVHRUGHFUHDVHRIWKHUDGLXVRIWKHRUELW




6ROXWLRQRITXHVWLRQ
D

VWPHWKRG
)RUHTXLOLEULXPZHKDYH)F )J1
ZKHUH1LVQRUPDOWRWKHVXUIDFH
5HVROYLQJLQWRKRUL]RQWDODQGYHUWLFDO
FRPSRQHQWVZHILQG

)URP

ZHILQG
ZKHUH

7KLVPHDQVWKDWDOWKRXJKUGHSHQGVRQ[DQG\WKHFKDQJHLQWKHIDFWRULQIURQWRI
[G[LVVRVOLJKWWKDWZHFDQWDNHLWWREHFRQVWDQW7KHVROXWLRQRI(T  LVWKHQDQ
HOOLSVH

DQGIURPWKLVLWIROORZVWKDW

QGPHWKRG
)RUDSRLQWPDVVRINJRQWKHVXUIDFH

7KHIRUPRIWKHVXUIDFHLVVXFKWKDW8SRW8NLQ FRQVWDQW)RUWKHHTXDWRU 0 


U UH DQGIRUWKHSROH 0 %U US ZHKDYH

7KXV

E

$VDFRQVHTXHQFHRIWKHVWDUTXDNHWKH
PRPHQWRILQHUWLDRIWKHFUXVW,PGHFUHDVHV
E\

)URPWKHFRQVHUYDWLRQRIDQJXODU
PRPHQWXPZHKDYH

$IWHUWKHLQWHUQDOIULFWLRQKDVHTXDOL]HGWKHDQJXODUYHORFLWLHVRIWKHFUXVWDQGWKH
FRUHZHKDYH

0DUNLQJEUHDNGRZQ
D

VWPHWKRG

H[SUHVVLRQVIRUWKHIRUFHV
HTXDWLRQIRUWKHVXUIDFH
HTXDWLRQRIHOOLSVH
IODWWHQLQJIDFWRU






QGPHWKRG

HQHUJ\HTXDWLRQ
IODWWHQLQJIDFWRU




FRQVHUYDWLRQRIDQJXODUPRPHQWXPIRUFUXVW
FRQVHUYDWLRQRIDQJXODUPRPHQWXPIRUFUXVWDQGFRUH
PRPHQWRILQHUWLDIRUDVSKHUH
UDWLRUFU






6ROXWLRQRITXHVWLRQ


7KHOLQHDULW\RIWKHSKRWRGLRGH

7KHOLQHDULW\RIWKHSKRWRGLRGHFDQEHFKHFNHGE\XVLQJWKHLQYHUVHVTXDUHODZEHWZHHQ
GLVWDQFHDQGLQWHQVLW\6XSSRVHWKDWWKHPHDVXUHGGLVWDQFHEHWZHHQWKH/('DQGWKH
ER[FRQWDLQLQJWKH 3'LV[7KHLQWHQVLW\RIWKHOLJKWIDOOLQJRQWKH3'VDWLVILHV

,IWKHLQWHQVLW\LVLQGHHGSURSRUWLRQDOWRWKHFXUUHQWIORZLQJWKURXJKWKH3'LWZLOODOVREH
SURSRUWLRQDOWRWKHYROWDJH9 [ PHDVXUHGDFURVVWKHUHVLVWRU5)URP  LWWKHQ
IROORZVWKDW

7RREWDLQWKHFRUUHFWYDOXHRI9 [ RQHVKRXOGVXEWUDFWIURPWKHPHDVXUHGYROWDJH9
WKHYROWDJH9WKDWRQHPHDVXUHVZKHQWKH/('LVWXUQHGRII EXWWKH/('ER[LVVWLOOLQ
SODFHLQIURQWRIWKH3' 
[ FP

9 9

9 9

L $

L $

>L [ L [ @ $































































































































3ORWWHGRQDJUDSKRQHILQGVDSHUIHFWVWUDLJKWOLQH



7KHOLJKWLQWHQVLW\DVDIXQFWLRQRIWKHHOHFWULFDOSRZHURIWKH/('

7KHSKRWRFXUUHQWL3'LVGHWHUPLQHGIURPWKHYROWDJH9RYHU5 067KHPHWHULWVHOI
KDVDQLQWHUQDOUHVLVWDQFHRI06LQWKHP9UDQJHDQG06LQWKHRWKHU
UDQJHV:HKDYHWKHQL3' 9UHVSL3' 9ZKHUH9LVLQYROWVDQGL3'LQ$
7KHFXUUHQWLQDPSqUHVWKURXJKWKH/('LVWKHYROWDJHRYHU5LQYROWVGLYLGHGE\
_3'__/('_
>[ FP@
9 9

9 9

LL $

L/(' $ 9/(' 9

3/(' : LL 3/('














































































7KHHIILFLHQF\LVSURSRUWLRQDOWR LL 3/(',QWKHJUDSKRI LL 3/('DJDLQVWL/('WKH


PD[LPDOHIILFLHQF\FRUUHVSRQGVWRL/(' $ 6HHILJXUH


'HWHUPLQDWLRQRIWKHPD[LPDOHIILFLHQF\

7KH/('HPLWVDFRQLFDOEHDPZLWKF\OLQGULFDOV\PPHWU\6XSSRVHZHPHDVXUHWKHOLJKW
LQWHQVLW\ZLWKD3'RIVHQVLWLYHDUHDGDWDGLVWDQFHULIURPWKHD[LVRIV\PPHWU\/HWWKH
LQWHQVLW\RIWKHOLJKWWKHUHEH0 UL WKHQZHKDYH

UL PP 9 9 9 9 LL UL [$P UL PP 9 9 9 9 LL UL [
$P

  

 

  

  

  

  

  

  

  

  

  

  

  

  

  

  

  

  

  

  

  

  

  

  

  

  


7KHHIILFLHQF\ 03/('

0DUNLQJEUHDNGRZQ
OLQHDULW\RIWKH3'
LQYHUVHVTXDUHODZ
QXPEHURIPHDVXULQJSRLQWV>!>!>!
GDUNFXUUHQW
FRUUHFWJUDSK






GHWHUPLQDWLRQRIFXUUHQWDWPD[LPDOHIILHQF\
SULQFLSOH
QXPEHURIPHDVXULQJSRLQWV>!>!>!
JUDSKHIILFLHQF\FXUUHQW
GHWHUPLQDWLRQRIFXUUHQWDWPD[LPDOHIILFLHQF\






GHWHUPLQDWLRQRIWKHPD[LPDOHIILFLHQF\
GHWHUPLQDWLRQRIWKHHPLWWHGOLJKWLQWHQVLW\

YLDHVWLPDWLRQRIWKHFRQHFURVVVHFWLRQ

YLDPHDVXUHPHQWRIWKHLQWHQVLW\GLVWULEXWLRQ

GHWHUPLQDWLRQRIWKHPD[LPXPHIILFLHQF\


6ROXWLRQRITXHVWLRQ


7KHRU\

/HW

WKHPRPHQWRILQHUWLDRIWKHGLVNEH
WKHPDVVRIWKHZHLJKW
WKHPRPHQWRIWKHIULFWLRQDOIRUFH
PDJQHWLFILHOGVWUHQJWK
WKHUDGLXVRIWKHD[OH
WKHPRPHQWRIWKHPDJQHWLFIRUFH

,
P
0I
%
U
0%

)RUWKHPRWLRQRIWKHURWDWLQJGLVNZHKDYH

:HVXSSRVHWKDW0ILVFRQVWDQWEXWQRWQHJOLJLEOH%HFDXVHWKHGLVNPRYHVLQWKH
PDJQHWLFILHOGHGG\FXUUHQWVDUHVHWXSLQWKHGLVN7KHPDJQLWXGHRIWKHVHFXUUHQWVLV
SURSRUWLRQDOWR%DQGWRWKHDQJXODUYHORFLW\7KH/RUHQW]IRUFHDVDUHVXOWRIWKHHGG\
FXUUHQWVDQGWKHPDJQHWLFILHOGLVWKXVSURSRUWLRQDOWRWKHVTXDUHRI%DQGWRWKHDQJXODU
YHORFLW\LH
6XEVWLWXWLQJWKLVLQWR(T  ZHILQG

$IWHUVRPHWLPHWKHGLVNZLOOUHDFKLWVILQDOFRQVWDQWDQJXODUYHORFLW\WKHDQJXODU
DFFHOHUDWLRQLVQRZ]HURDQGIRUWKHILQDOYHORFLW\YHZHILQG
7KHILQDOFRQVWDQWYHORFLW\LVWKXVDOLQHDUIXQFWLRQRIP


7KHH[SHULPHQW

7KHILQDOFRQVWDQWVSHHGLVGHWHUPLQHGE\PHDVXULQJWKHWLPHWDNHQWRIDOOWKHODVWFP
>WKLVLVWKHZLGWKRIDVKHHWRISDSHU@
,QWKHILUVWSODFHLWLVQHFHVVDU\WRFKHFNWKDWWKHILQDOVSHHGKDVEHHQUHDFKHG7KLVLV
GRQHE\DOORZLQJWKHZHLJKWWRIDOORYHUGLIIHUHQWKHLJKWV,WLVFOHDUWKDWZLWKWKHZHDNHU
PDJQHWWKHQHFHVVDU\KHLJKWEHIRUHWKHFRQVWDQWVSHHGLVDWWDLQHGZLOOEHODUJHU
0HDVXUHPHQWVIRUWKHZHDNPDJQHWV\VWHP

WLPHWDNHQWRIDOO
KHLJKW P

VPDOOHUZHLJKW

ODUJHUZHLJKW











 V
 V
 V
 V
 V
 V
 V



 V
 V
 V
 V
 V
 V
 V
 V
 V



)LQDOFRQVWDQWVSHHGPHDVXUHPHQWVIRUERWKPDJQHWV\VWHPVDQGIRUVHYHUDO
FKRLFHVRIZHLJKW
0HDVXUHPHQWVIRUWKHZHDNPDJQHW

ZHLJKW 7 V

7 V

7 V

7 V

7! V

Y! PV

VPDOO
ODUJH
ERWK

























0HDVXUHPHQWVIRUWKHVWURQJPDJQHW
ZHLJKW 7 V

7 V

7 V

7 V

7! V

Y! PV

VPDOO
ODUJH
ERWK



























'LVFXVVLRQRIWKHUHVXOWV
$JUDSKEHWZHHQYHDQGWKHZHLJKWVKRXOGEHPDGH
)URP(T  ZHREVHUYHWKDW
ERWKVWUDLJKWOLQHVVKRXOGLQWHUVHFWRQWKHKRUL]RQWDOD[LV
IURPWKHVTXDUHURRWRIWKHUDWLRRIWKHVORSHVZHKDYHLPPHGLDWHO\WKH
UDWLRRIWKHPDJQHWLF ILHOGVWUHQJWKV
)RUWKHDERYHPHDVXUHPHQWVZHILQG

0DUNLQJ%UHDNGRZQ


(T 





,QYHVWLJDWLRQRIWKHUDQJHLQZKLFKWKHVSHHGLVFRQVWDQW



1XPEHURIWLPLQJPHDVXUHPHQWV>@
(UURUHVWLPDWLRQ




JUDSK






TXDOLW\
WKHOLQHVLQWHUVHFWHDFKRWKHURQWKHPDVVD[LV
FDOFXODWLRQRI%%
(UURUFDOFXODWLRQ

THEORETICAL

PROBLEMS

Problem 1
The figure 1.1 shows a solid, homogeneous ball radius R. Before falling to the floor its center of mass is
at rest, but the ball is spinning with angular velocity 0 about a horizontal axis through its center. The
lowest point of the ball is at a height h above the floor.

When released, the ball falls under gravity, and rebounds to a new height such that its lowest point is now
ah above the floor. The deformation of the ball and the floor on impact may be considered negligible.
Ignore the presence of the air. The impact time, although, is finite.
The mass of the ball is m, the acceleration due the gravity is g, the dynamic coefficient of friction between
the ball and the floor is k, and the moment of inertia of the ball about the given axis is:
I=

2mR 2
5

You are required to consider two situations, in the first, the ball slips during the entire impact time, and in
the second the slipping stops before the end of the impact time.
Situation I: slipping throughout the impact.
Find:
a) tan , where is the rebound angle indicated in the diagram;
b) the horizontal distance traveled in flight between the first and second impacts;
c) the minimum value of 0 for this situations.
Situation II: slipping for part of the impacts.
Find, again:
a) tan ;
b) the horizontal distance traveled in flight between the first and second impacts.
Taking both of the above situations into account, sketch the variation of tan with 0.
Problem 2
In a square loop with a side length L, a large number of balls of negligible radius and each with a charge q
are moving at a speed u with a constant separation a between them, as seen from a frame of reference that
is fixed with respect to the loop. The balls are arranged on the loop like the beads on a necklace, L being
much greater than a, as indicated in the figure 2.1. The no conducting wire forming
the loop has a homogeneous charge density per unit length in the in the frame of the loop. Its total charge
is equal and opposite to the total charge of the balls in that frame.

Consider the situation in which the loop moves with velocity v parallel to its side AB (fig. 2.1) through a
homogeneous electric field of strength E which is perpendicular to the loop velocity and makes an angle
with the plane of the loop.

Taking into account relativistic effects, calculate the following magnitudes in the frame of reference of an
observer who sees the loop moving with velocity v:
a) The spacing between the balls on each of the side of the loop, aAB , aBC , aCD , y aDA.
b) The value of the net charge of the loop plus balls on each of the side of the loop: QAB , QBC , QCD y, QDA
c) The modulus M of the electrically produced torque tending to rotate the system of the loop and the
balls.
d) The energy W due to the interaction of the system, consisting of the loop and the balls with the electric
field.
All the answers should be given in terms of quantities specified in the problem.
Note. The electric charge of an isolated object is independent of the frame of reference in which the
measurements takes place. Any electromagnetic radiation effects should be ignored.
Some formulae of special relativity
Consider a reference frame S moving with velocity V with reference to another reference frame S. The
axes of the frames are parallel, and their origins coincide a t = 0. V is directed along the positive direction
of the x axis.
Relativistic sum of velocities
If a particle is moving with velocity u in the x direction , as measured in S, the velocity of the particle
measured in S is given by:

u=

u + V
uV
1+ 2
c

Relativistic Contraction
If an object at rest in frame S has length L0 in the x-direction, an observer in frame S (moving at velocity
V in the x-direction} will measure its length to be:
L = L0 1

v2
c2

Problem 3 Cooling Atoms by laser


To study the properties of isolated atoms with a high degree of precision they must be kept almost at rest
for a length of time. A method has recently been developed to do this. It is called laser cooling and is
illustrated by the problem below.
In a vacuum chamber a well collimated beam of Na23 atoms (coming from the evaporation of a sample at
103 K) is illuminated head-on with a high intensity laser beam (fig. 3.1). The frequency of laser is chosen
so there will be resonant absorption of a photon by those atoms whose velocity is v0. When the light is
absorbed, these atoms are exited to the first energy level, which has a mean value E above the ground
state and uncertainty of (fig. 3.2).

For this process, the atoms decrease in velocity v1 is given by v1 = v1 v0. Light is then emitted by the
atom as it returns to its ground state. The atoms velocity changes by v = v1 v1 and its direction of
motion changes by an angle (fig. 3.3).

This sequence of absorption and emission takes place many times until the velocity of the atoms has
decreased by a given amount v such that resonant absorption of light at frequency v no longer occurs. It
is then necessary to change the frequency of laser so as to maintain resonant absorption. The atoms
moving at the new velocity are further slowed down until some of them have a velocity close to zero.
As first approximation we may ignore any atomic interaction processes apart from the spontaneous
absorption and emission light described above.
Furthermore, we may assume the laser to be so intense that the atoms spend practically no time in the
ground state.

Questions
a) Find the laser frequency needed ensure the resonant absorption of the light by those atoms whose
kinetic energy of the atoms inside the region behind the collimator. Also find the reduction in the
velocity of these atoms, v1, after the absorption process.
b) Light of the frequency calculated in question a) is absorbed by atoms which velocities lie within a
range v0. Calculate this velocity range.

c) When an atom emits light, its direction of motion changes by from initial direction. Calculate .
d) Find the maximum possible velocity decrease v for a given frequency.
e) What is the approximate number N of absorption-emission events necessary to reduce the velocity of
an atom from is initial value vo
-found in question a) above- almost to zero? Assume the atom travels in a straight line.
f)Find the time t that the process in question e takes. Calculate the distance S an atom travels in this
time.
Data
E = 3,3610-19 J
= 7,010-27 J
c = 3108 ms-1
mp = 1,6710-27 kg
h = 6,6210-34 Js
k = 1,3810-23 JK-1
where c is speed of light, h is Plancks constant, k is the Boltzmann constant, and mp is the mass of
proton.

T H E O R E T I C A L P R O B L E M S.

SOLUTIONS

Solution Problem 1
a) Calculation of the velocity at the instant before impact
Equating the potential gravitational energy to the kinetic energy at the instant before impact we can arrive
at the pre-impact velocity v0:

mv02
mgh =
2

(1)

from which we may solve for v0 as follows:


V0 =

2 gh

(2)

b) Calculation of the vertical component of the velocity at the instant after impact
Let v2x and v2y be the horizontal and vertical components, respectively, of the velocity of the mass center
an instant after impact. The height attained in the vertical direction will be h and then:

v 22y = 2gh

(3)

from which, in terms of (or the restitution coefficient c =

):

v2y = 2gh = cv0


(4)
c) General equations for the variations of linear and angular momenta in the time interval of the
Impact

Considering that the linear impulse of the forces is equal to the variation of the linear momentum and
that the angular impulse of the torques is equal to the variation of the angular momentum, we have:
t2

Iy =

N (t )dt

= mv0 + mv2y = m (1 + c)

2 gh

(5)

t1
t2

Ix =

f (t )dt
r

t1

= mv2x

(6)

t2

t2

Rf (t )dt

I =

= R

t1

f (t )dt
r

= I (o 2)

(7)

t1

Where Ix, Iy and I are the linear and angular impulses of the acting forces and 2 is the angular velocity
after impact. The times t1 and t2 correspond to the beginning and end of impact.
Variants
At the beginning of the impact the ball will always be sliding because it has a certain angular velocity 0.
There are, then, two possibilities:
I. The entire impact takes place without the friction being able to spin the ball enough for it to stop at the
contact point and go into pure rolling motion.
II. For a certain time t (t1, t2), the point that comes into contact with the floor has a velocity equal to
zero and from that moment the friction is zero. Let us look at each case independently.
Case I
In this variant, during the entire moment of impact, the ball is sliding and the friction relates to the normal
force as:
fr = kN(t)
(8)
Substituting (8) in relations (6) and (7), and using (5), we find that:
t2

Ix = k

N (t )dt

= kIy = k (1 + c )

2 gh = mv2x

(9)

2 gh = I( 0 2)

(10)

t1

and:
t1

I = R k

N (t )dt

= R k m(1 + c)

t1

which can give us the horizontal component of the velocity v2x and the final angular velocity in the form:
V2x = k (1 + c)

2 = 0

2 gh

k mR(1 + c )
I

(11)

2 gh

(12)

With this we have all the basic magnitudes in terms of data. The range of validity of the solution under
consideration may be obtained from (11) and (12). This solution will be valid whenever at the end of the
impact the contact point has a velocity in the direction of the negative x. That is, if:

2R > v2x
0

0 >

k mR(1 + c )
I

k 2 gh
R

2 gh >

k (1 + c )
R

2 gh

mR 2

(1 + c)
+ 1
I

(13)

so, for angular velocities below this value, the solution is not valid.
Case II
In this case, rolling is attained for a time t between the initial time t1 and the final time t2 of the impact.
Then the following relationship should exist between the horizontal component of the velocity v2x and the
final angular velocity:
2R = v2x
(14)
Substituting (14) and (6) in (7), we get that:

mRv2x = I 0

v2 x

(15)

which can be solved for the final values:


V2x =

and:

2 =

I 0

I
mR +
R

I 0 R
2
=
0 R
2
7
mR + I

I 0
2
=
0
2
7
mR + I

(16)

(17)

Calculation of the tangents of the angles


Case I
For tan we have, from (4) and (11), that:

k (1 + c ) 2 gh
v2 x
(1 + c)
=
= k
v2 y
c
c 2 gh
(1 + c)
tan = k
c
tan =

(18)

i.e., the angle is independent of 0.

Case II
Here (4) and (16) determine for tan that:
tan =
tan =

I 0 R
I 0 R
v2 x
1
=
=
2
v2 y
I + mR c 2 gh ( I + mR 2 )c 2 gh

2 0 R

(19)

7c 2 gh

then (18) and (19) give the solution (fig. 1.3).

We see that does not depend on o if 0 > 0 min; where 0 min is given as:

k (1 + c ) 2 gh 1 +
o min =

mR 2

7 k (1 + c ) 2 gh
2R

o min =

(20)

Calculation of the distance to the second point of impact


Case I
The rising and falling time of the ball is:
tv = 2

v2 y
g

2c 2 gh
g

= 2c

2h
g

(21)

The distance to be found, then, is;


dI = v2xtv = k (1+ c)

2 gh 2c

2h
g

d1 = 4 k(1+ c ) ch
which is independent of 0.

(22)

Case II
In this case, the rising and falling time of the ball will be the one given in (21). Thus the distance we are
trying to find may be calculated by multiplying tv by the velocity v2x so that:

dII = v2xtv =

dII =

I 0
2h
2c
2
g
mR + I

2 0 Rc 2h
5
g
1+
2

4 2h
c
R 0
7
g

Thus, the distance to the second point of impact of the ball increases linearly with 0.
Marking Code
The point value of each of the sections is:
1.a
2 points
1.b
1.5 points
1.c
2 points
2.a
2.b
3

2 points
1.5 points
1 point

Solution Problem 2
Question a:
Lets call S the lab (observer) frame of reference associated with the observer that sees the loop moving
with velocity v; S to the loop frame of reference (the x axis of this system will be taken in the same
r
direction as v ; y in the direction of side DA and z axis, perpendicular to the plane of the loop). The
axes of S are parallel to those of S and the origins of both systems coincide at t = 0.
1. Side AB
''
S AB
will be a reference frame where the moving balls of side AB are at rest. Its axes are parallel

'

''

'

to those of S and S . S has a velocity u with respect to S .


''

According to the Lorentz contraction, the distance a, between adjacent balls of AB, measured in S , is:

ar =

(1)

u2
1 2
c

(This result is valid for the distance between two adjacent balls that are in one of any sides, if a, is
measured in the frame of reference in which they are at rest).
Due to the relativistic sum of velocities, an observer in S sees the balls moving in AB with velocity:

u AB =

v+u
uv
1+ 2
c

(2)

So, because of Lorentz contraction, this observer will see the following distance between balls:

a AB = 1

2
u AB
ar
c2

(3)

Substituting (1) and (2) in (3)

v2
t2 a
=
uv
1+ 2
c
1

a AB

(4)

2. Side CD
For the observer in S, the speed of balls in CD is:

u CD =

vu
uv
1 2
c

(5)

From the Lorentz contraction:

aCD = 1

2
u CD
ar
c2

(6)

Substituting (1) and (5) in (6) we obtain:

v2
c2
=
a
uv
1 2
c
1

aCD

(7)

3. Side DA
In system S, at time t o , let a ball be at x1 = y1 = z1 = 0 . At the same time the nearest neighbour to
'

'

'

'

this ball will be in the position x 2 = 0, y 2 = a , z 2 = 0.


The space-time coordinates of this balls, referred to system S, are given by the Lorentz transformation:
'

x=

1
v2
1 2
c

'

( x'+ vt ' )

y=y
z=z

t=

'

(8)

x ' v'
t '+ 2
c
v2
1+ 2
c
1

Accordingly, we have for the first ball in S:

x1 =

vt o' ; y1=0; z1=0; t 1 =

v
1 2
c

1
2

v
1 2
c

t '0

(9)

(9)
And for the second:

x2 =

vt 'o ; y 2 = a; z 2 = 0; t 2 =

v
1 2
c

1
2

v
1 2
c

t 'o

As t1= t2, the distance between two balls in S will be given by:
aDA=(x2-x1)2 + (y2 y1)2 + (z2-z1)2
So:
aAD=a
4. Side BC
If we repeat the same procedure as above, we can obtain that:
aBC=a

(10)

(11)
(12)

(13)
Question b:
The charge of the wire forming any of the sides, in the frame of reference associated with the loop can be
calculated as:

L
Q wire = q
a

(14)

Because L/a is the number of balls in that side. Due to the fact that the charge in invariant, the same
charge can be measured in each side of the wire in the lab (observer) frame of reference.
1. Side AB
The charge corresponding to balls in side AB is, in the lab frame of reference:

Q AB, balls

v2
L 1 2
c
=
q
a AB

(15)

This is obtained from the multiplication of the number of balls in that side multiplied by the (invariant)
charge of one ball. The numerator of the first factor in the right side of equation (15) is the contracted
distance measured by the observer and the denominator is the spacing between balls in that side.
Replacing in (15) equation (4), we obtain:

1 + uv Lq
Q AB, balls = 2
c a

(16)

Adding (14) and (16) we obtain for the total charge of this side:

Q AB =

uv L
q
c2 a

(17)

2. Side CD
Following the same procedure we have that:

1
Q CD, balls =

v2
c2

a CD

uv Lq
q = 1 2
c a

(18)

And adding (14) and (18) we obtain:

Q CD =

uv L
q
c2 a

(19)

The length of these sides measured by the observer in S is L and the distance between balls is a, so:

Q BC, balls = QDA , balls =

Lq
a

Adding (14) and (20) we obtain:

(20)

QBC = 0
QDA=0
Question c:
There is electric force acting into the side AB equal to:

(21.1)
(21.2)

uv L
F AB = Q AB E = 2 q E
c a

(22)

and the electric force acting into the side CD is:

uv L
F CD = Q CD E = 2 q E
c a

(23)

FCD and F, form a force pair. So, from the expression for the torque for a force pair we have that (fig. 2.2):

M = F AB L sin

(24)

And finally:

M=

uv L2
q E sin
c2 a

(25)

Fig 2.2
Question d:
Lets call VAB and VCD the electrostatic in the points of sides AB and CD respectively. Then:
W=VABQAB + VCDQCD

(26)

Lets fix cero potential (V=0) in a plane perpendicular to E and in an arbitrary distance R from side AB
(fig. 2.3).

Figure 2.3
Then:
W=-ERQAB E(R+Lcos)QCD
But QCD=-QAB, so:
W=-ELQABcos

(27)
(28)

Substituting (17) in (28) we obtain:

W=

uvL2 qE
cos
c 2a

(29)

Marking Code
Grading for questions will be as follows:
a)4,5 points.
b)2,0 points.
c)1,5 points.
d)2,0 points.
These points are distributed in questions in the following way:
Question a:
1. Obtaining expressions (4) and (7) correctly: 3,0 points.
Only one of them correct: 2,0 points.
2. Obtaining expressions (12) and (13) correctly including the necessary calculations to arrive to this
results: 1,5 points.
Only one of them correct: 1,0 points.
If the necessary calculations are not present: 0,8 point for both (12) and (13) correct; 0,5 points for only
one of them correct.
Question b:
1. Obtaining expressions (17) and (19) correctly: 1,0 point.
Only one of them correct: 1,0 point.
2. Obtaining expressions (21.1) and (21.2) correctly: 0,5 point.
Only one them correct: 0,5 point.
Question d:
1. Obtaining expression (29) correctly: 2,0 points.
When the modulus of a vector is not present where necessary, the student will loose 0,2 points. When the
modulus of q is not present where necessary the student will loose 0,1 points.
Solution Problem 3
Question a:
The velocity vo of the atoms whose kinetic energy is the mean of the atoms on issuing from the collimator
is given is given by:

1
3
3kT
mv o2 = kT v o =
2
2
m

(1)

3 1,38 10 23 10 3
m/s
23 1,67 10 27
vo 1,04103 m/s because:
m 23 mp
(2)
Since this velocity is much smaller than c, vo<< c, we may disregard relativistic effects.
Light is made up of photons with energy h and momentum h/c.
In the reference system of the laboratory, the energy and momentum conservation laws
applied to the absorption process imply that:
1
1
h
h
mv o2 + h = mv 12 + E; mv o
= mv 1 v 1 = v 1 v o =
2
2
c
mc
1
1
m( v 12 v o2 ) = h E m( v 1 + v o )( v 1 v o ) = h E
2
2
h/c<<mvo. Then v1 vo and this implies mvov1 = h = E, where we assume that
v1 + vo 2vo
Combining these expressions:
vo =

E
= h
v
1+ o
c
and:
E
v 1 =
mc

(3)

1
(4)
vo
1+
c
And substituting the numerical values:
5,01014 Hz v1 -3,010-2 m/s
If we had analyzed the problem in the reference system that moves with regard to the
laboratory at a velocity vo, we would have that:
1
m( v 1 v 2 ) 2 + E = h
2
'
Where =
is the frequency of the photons in the laboratory
vo
1+
c
system. Disregarding v 12 we get the same two equations above.
The approximations are justifiable because:
v 1
10-4

vo
Then v1 + vo = 2vo - v1 2vo
Question b:
For a fixed :
E

v o = c
1
(5)
h
if E has an uncertainty , vo would have an uncertainty:
v
c 1 + o
c c
c
v o =
=

= 6,25 m/s
(6)
h
E
E
so the photons are absorbed by the atoms which velocities are in the interval
v
v

vo o , vo + o
2
2

Question c:
The energy and momentum conservation laws imply that:
1
1
mv 12 + E = mv '12 +h'
2
2
( is the frequency of emitted photon)
h '
mv 1 = mv '1 cos +
cos
c
h '
0 = mv '1 sin
sin
c

The deviation of the atom will be greatest when = , then:


2

mv 1 = mv '1 cos m ;

h'
h'
= mv '1 sin m tan m =
c
mv 1c

since :
E
tan m
mv 1c
E
m = arctg
m 5 10 5 rad
mvc

(7)
(8)

Question d:
As the velocity of the atoms decreases, the frequency needed for resonant absorption
increases according to:
E
= h
v
1+ o
c
When the velocity is vo = v, absorption will still be possible in the lower part of the
level if:

E
c v o
E
2
(9)
h =
=
v =
1+

v o v
vo
2E
c
1+
1+
c
c
v = 3,12 m/s
Question e:
If each absorption-emission event varies the velocity as v 1

E
, decreasing velocity
mc

from vo to almost zero would require N events, where:


v
mcv o
N= o
N 3,56 10 4
v 1
E
Question f:
If absorption is instantaneous, the elapsed time is determined by the spontaneous
h
emission. The atom remains in the excited state for a certain time, = , then:

mchv
Nh
o
t = N =
=
t 3,37 10 9 s

E
The distance covered in that time is S=vot/2. Assuming that the motion is uniformly
slowed down:
1
S = mchv 2o E S 1,75 m
2
Marking Code
a)Finding

b)

vo

v1
vo

1 pt
1 pt
1 pt
1,5 pt

Total 3 pt

Total 1,5 pt

c)
d)
e)
f)

m
v
N
t
S

1,5 pt
1 pt
1 pt
1 pt
1 pt

Total 1,5 pt
Total 1 pt
Total 1 pt
Total 2 pt
____________
Overall total 10 pts
We suggest in all cases: 0,75 for the formula; 0,25 for the numeral operations.
EXPERIMENTAL

PR O B L E M

Problem
Inside a black box provided with three terminals labeled A, B and C, there are three electric components
of different nature. The components could be any of the following types: batteries, resistors larger then
100 ohm, capacitors larger than 1 microfarad and semiconductor diodes.
a)Determine what types of components are inside the black box and its relative position to terminal A, B
and C. Draw the exploring circuits used in the determination, including those used to discard circuits
with similar behaviour
b)If a battery was present, determine its electromotive force. Draw the experimental circuit used.
c)If a resistor was present, determine its value. Draw the experimental circuit used.
d)If a capacitor was present, determine its value. Draw the experimental circuit used.
e)If a diode was present, determine Vo and Vr, where Vo the forward bias threshold voltage and Vr is the
reverse bias breakdown voltage.
f)Estimate, for each measured value, the error limits.
The following equipments and devices are available for your use:
1 back box with three terminals labeled A, B and C;
1 variable DC power supply;
2 Polytest 1 W multimeters;
10 connection cables;
2 patching boards;
1 100 k, 5 % value resistor;
1 10 k, 5 % value resistor;
1 1 k, 5 % value resistor;
1 100 F, 20 % value capacitor;
1 chronometer;
2 paper sheets;
1 square ruler;
1 interruptor.
Voltmeter internal resistance.
Scale
Value in k
0-1 V
3,2
1%
0-3 V
10
1%
0-10 V
32
1%
0-20 V
64
1%
0-60 V
200
1%
Ammeter internal resistance.
Scale
Value in
0-0,3 mA
1 000 1 %
0-1 mA
263 1 %
0-3 mA
94 1 %
0-20 mA
30,4 1 %
0-30 mA
9,84 1 %
0-100 mA
3,09 1 %
0-300 mA
0,99 1 %
0-1 mA
0,31 1 %

Notice: Do not use the Polystes 1 W as an ohmmeter. Protect your circuit against large currents, and do
not use currents larger than 20 mA.
Give your results by means of tables or plots.
When drawing the circuits, use the following symbols:

EXPERIMENTAL

PR O B L E M. S O L U T I O N

Solution Problem
Since a battery could be present, the first test should be intended to detect it. To do that, the voltage drops
Vab, Vac and Vbc should be measured using a voltmeter. This test will show that no batteries are present.
Next, a testing circuit as shown in figure 4.1 should be used.

By means of this circuit, the electric conduction between a pair of terminals should be tested, marking all
permutations and reversing the polarity. Resistor R1 is included to prevent a large current across the
diode. One conclusion is that between A and C there is a diode and a resistor in series, although its
current position is still unknown. The other conclusion is that a capacitor is tighted to terminal B. To
determine the actual circuit topology, further transient experiments have to be conducted.
In this way, it is concluded that the actual circuit inside the black box is that shown in figure 4.2.

The best procedure for the resistor value determination is to plot a set of voltage and current values
measured between A and C. Figure 4.3 shows the resulting plot. Extrapolating both linear regions, the
values of Vo and Vz are obtained and the resistor value equals the reciprocal of the slope.
Similar, the best method to measure the capacitor value is to build a testing circuit as shown in figure 4.4.
The current is adjusted to full scale and then, the switch is opened.
The time needed by the current to drop to its half value is measured. Applying the formulae t = RCIn(2),
the value of C is obtained.

Marking Code
1. Determination of circuit topology: 8 points.
1.1 For discarding the presence of a battery: 1 point.
1.2 For drawing the exploring circuit which determine the circuit topology in a unique way: 7 points.
2. Resistor and diode parameters value measurement: 8 points.
2.1 For drawing the measuring circuit: 2 points.
2.2 Error limits calculation: 3 points.
2.3 Result: 3 points.
2.3.1 Coarse method: 2 points.
2.3.2 Graphic method: 3 points.
3. Capacitor value measurement: 4 points.
3.1 For drawing the measuring circuit: 2 points.
3.2 Error limits calculations: 2 points.

THE EXAMINATION
XXV INTERNATIONAL PHYSICS OLYMPIAD
BEIJING, P ERPLES REPUBLIC CHINA
THEORETICAL COMPETITION
July 13, 1994
Time available: 5 hours
READ THIS FIRST!
INSTRUCTIONS:
1. Use only the ball pen provided.
2. Your graphs should be drawn on the answer sheets attached to the problem.
3. Your solutions should be written on the sheets of paper attached to the problems.
4. Write at the top of the first page of each problem:
The total number of pages in your solution to the problem
Your name and code number

Theoretical Problem 1

RELATIVISTIC PARTICLE
In the theory of special relativity the relation between energy E and momentum P
or a free particle with rest mass m0 is

E=

p 2 c 2 + m02 c 4 = mc 2

When such a particle is subject to a conservative force, the total energy of the
particle, which is the sum of

p 2 c 2 + m02 c 4 and the potential energy, is conserved. If

the energy of the particle is very high, the rest energy of the particle can be ignored
(such a particle is called an ultra relativistic particle).
1) consider the one dimensional motion of a very high energy particle (in which
rest energy can be neglected) subject to an attractive central force of constant
magnitude f. Suppose the particle is located at the centre of force with initial
momentum p0 at time t=0. Describe the motion of the particle by separately
plotting, for at least one period of the motion: x against time t, and momentum
p against space coordinate x. Specify the coordinates of the turning points in
terms of given parameters p0 and f. Indicate, with arrows, the direction of the
progress of the mothon in the (p, x) diagram. There may be short intervals of
time during which the particle is not ultrarelativistic. However, these should be
neglected.
Use Answer Sheet 1.
2) A meson is a particle made up of two quarks. The rest mass M of the meson is
equal to the total energy of the two-quark system divided by c2.
Consider a one--dimensional model for a meson at rest, in which the two
quarks are assumed to move along the x-axis and attract each other with a force
of constant magnitude f It is assumed they can pass through each other freely.
For analysis of the high energy motion of the quarks the rest mass of the quarks
can be neglected. At time t=0 the two quarks are both at x=0. Show separately
the motion of the two quarks graphically by a (x, t) diagram and a (p, x)
diagram, specify the coordinates of the turning points in terms of M and f,
indicate the direction of the process in your (p, x)

diagram, and determine the

maximum distance between the two quarks.


Use Answer Sheet 2.
3) The reference frame used in part 2 will be referred to as frame S, the Lab frame,
referred to as S, moves in the negative x-direction with a constant velocity
v=0.6c. the coordinates in the two reference frames are so chosen that the point
2

x=0 in S coincides with the point x = 0 in S at time t = t = 0 . Plot the


motion of the two quarks graphically in a ( x , t ) diagram. Specify the
coordinates of the turning points in terms of M, f and c, and determine the
maximum distance between the two quarks observed in Lab frame S .
Use Answer Sheet 3.
The coordinates of particle observed in reference frames S and S are related
by the Lorentz transformation
x = ( x + ct )

t = (t + c )

where = v / c , = 1 / 1 2 and v is the velocity of frame S moving


relative to the frame S .
4) For a meson with rest energy Mc2=140 MeV and velocity 0.60c relative to the
Lab frame S , determine its energy E in the Lab Frame S .
ANSWER SHEET 1

ANSWER SHEET 2

1)

2)

x1, x2

p1

p2

x1

Quark1

x2
O

Quark2

The maximum distance between


the two quarks is d=

ANSWER SHEET 3
3)

The maximum distance between the


two quarks observed in Sframe is
d=

x1, x2

t
O
Theoretical Problem 1Solution
1) 1a. Taking the force center as the origin of the space coordinate x and the zero
potential point, the potential energy of the particle is
U ( x) = f | x |

(1)

The total energy is

W =

p 2 c 2 + m02 c 4 + f | x | .

1b. Neglecting the rest energy, we get


W =| p | c + f | x | ,

(2)

Since W is conserved throughout the motion, so we have


W =| p | c + f | x |= p 0 c ,

(3)

Let the x axis be in the direction of the initial momentum of the particle,
pc + fx = p 0 c

when
when
when
when

pc + fx = p 0 c
pc fx = p 0 c
pc fx = p 0 c

x > 0,
x > 0,
x < 0,
x < 0,

p > 0;
p < 0;

p > 0;
p < 0.

(4)

The maximum distance of the particle from the origin, let it be L, corresponds to p=0.
It is
L = p0 c / f .
1c. From Eq. 3 and Newtons law
f , x > 0;
dp
=F =
x < 0;
dt
f,

(5)

we can get the speed of the particle as


dx
c dp
=
= c,
dt
f dt
4

(6)

i.e. the particle with very high energy always moves with the speed of light except that
it is in the region extremely close to the points x = L . The time for the particle to
move from origin to the point x = L , let it be denoted by , is

= L / c = p0 / f .
So the particle moves to and for between x = L and x = L with speed c and period
4 = 4 p 0 / f . The relation between x and t is
0 t
t 2 ,

2 t 3 ,
3 t 4 ,

x = ct ,
x = 2 L ct ,
x = 2 L ct ,
x = ct 4 L,

(7)

The required answer is thus as given in Fig. 1 and Fig. 2.


x
L

p
A

p0
B

x
O

-L

-L

C
= p0/f
L= p0/f

-p0

Fig. 1

L=p0c/f

Fig. 2

2) The total energy of the two-quark system can be expressed as


Mc 2 =| p1 | c + | p 2 | c + f | x1 x 2 | ,

(8)

where x1 , x 2 are the position coordinates and p1 , p 2 are the momenta of quark 1
and quark 2 respectively. For the rest meson, the total momentum of the two quarks is
zero and the two quarks move symmetrically in opposite directions, we have

p = p1 + p 2 = 0 ,

p1 = p 2 ,

x1 = x 2 .

(9)

Let p0 denote the momentum of the quark 1 when it is at x=0, then we have
Mc 2 = 2 p 0 c

p 0 = Mc / 2

or

(10)

From Eq. 8, 9 and 10, the half of the total energy can be expressed in terms of p1 and

x1 of quark 1:
p 0 c =| p1 | c + f | x1 | ,

(11)

just as though it is a one particle problem as in part 1 (Eq. 3) with initial momentum
5

p 0 = Mc / 2 . From the answer in part 1 we get the (x, t) diagram and (p, x) diagram of
the motion of quark 1 as shown in Figs. 3 and 4. For quark 2 the situation is similar
except that the signs are reversed for both x and p; its (x, t) and (p, x) diagrams are
shown in Figs. 3 and 4.
The maximum distance between the two quarks as seen from Fig. 3 is
d = 2 L = 2 p 0 c / f = Mc 2 / f .
x1: solid line
x2: dash line

x1x2
A

E
F

O
-L

(12)

= Mc/2f
L=Mc2/2f

Fig. 3
p1

p2

p0

p0
x1

-L

O
-p0

x2
-L

L=Mc2/2f
P0=Mc/2

-p0

Fig. 4a

Quark1

Fig. 4b

Quark2

3) The reference frame S moves with a constant velocity V=0.6c relative to the Lab
frame S in the x axis direction, and the origins of the two frames are coincident at
the beginning ( t = t = 0 ). The Lorentz transformation between these two frames is
given by:
x = ( x + ct ),
t = (t + x / c),

(13)

where = V / c , and = 1 / 1 2 . With V = 0.6c , we have = 3 / 5 , and

= 5 / 4 . Since the Lorenta transformation is linear, a straight line in the (x, t) diagram
6

transforms into a straight line the ( x , t ) diagram, thus we need only to calculate the
coordinates of the turning points in the frame S .
For quark 1, the coordinates of the turning points in the frames S and S are as
follows:
Frame

x1

t1

Frame S
x1 = ( x1 + ct1 )

t1 = (t1 + x1 / e)
5
3
t1 + x1 / c
4
4
0

5
3
x1 + ct1
4
4
0

(1 + ) L = 2 L

(3 1) L = L

(3 ) = 3

4L = 3L

4 = 5

2L =

(1 + ) = 2

3
L
2

5
2 =
2

where L = p 0 c / f = Mc 2 / 2 f , = p 0 / f = Mc / 2 f .
For quark 2, we have
Frame

x2

t2

Frame S

x 2 = ( x 2 + ct 2 )

1
(1 ) L = L
2
3
2L = L
2
7
(3 + 1) L = L
2

5
3
t 2 + x2 / c
4
4
0
1
(1 ) =
2
5
2 =
2
9
(3 + ) =
2

4L = 3L

4 = 5

5
3
x 2 + ct 2
4
4
0

t 2 = (t 2 + x 2 / c)
=

With the above results, the ( x , t ) diagrams of the two quarks are shown in Fig. 5.
The equations of the straight lines OA and OB are:

x1 (t ) = ct ;

0 t (1 + ) = 2 ;

(14a)

x 2 (t ) = ct ;

1
0 t (1 ) =
2

(14b)

1
The distance between the two quarks attains its maximum d when t = , thus we
2
have maximum distance

Mc 2
.
d = 2c (1 ) = 2 (1 ) L =
2f

(15)

Fig. 5
4) It is given the meson moves with velocity V=0.6 crelative to the Lab frame, its
energy measured in the Lab frame is
E =

Mc 2
1

1
140 = 175 MeV.
0.8

Grading Scheme
Part 1 2 points, distributed as follows:
0.4 point for the shape of x(t) in Fig. 1;
0.3 point for 4 equal intervals in Fig. 1;
(0.3 for correct derivation of the formula only)
0.1 each for the coordinates of the turning points A and C, 0.4 point in total;
0.4 point for the shape of p(x) in fig. 2; (0.2 for correct derivation only)
0.1 each for specification of p 0 , L = p 0 c / f , p 0 , L and arrows, 0.5 point
in total.
(0.05 each for correct calculations of coordinate of turning points only).
Part 2 4 points, distributed as follows:
0.6 each for the shape of x1 (t ) and x 2 (t ) , 1.2 points in total;
0.1 each for the coordinates of the turning points A, B, D and E in Fig. 3, 0.8 point
in total;
8

0.3 each for the shape of p1 ( x1 ) and p 2 ( x 2 ) , 0.6 point in total;


0.1 each for p 0 = Mc / 2 , L = Mc 2 / 2 f , p 0 , L and arrows in Fig. 4a and
Fig. 4b, 1 point in total;
0.4 point for d = Mc 2 / f
Part 3 3 point, distributed as follows:
0.8 each for the shape of x1 (t ) and x 2 (t ) , 1.6 points in total;
0.1 each for the coordinates of the turning points A, B, D and E in Fig. 5, 0.8 point
in total; (0.05 each for correct calculations of coordinate of turning points
only).
0.6 point for d = Mc 2 / 2 f .
Part 4 1 point (0.5 point for the calculation formula; 0.5 point for the numerical value
and units)

Theoretical Problem 2
SUPERCONDUCTING MAGNET
Super conducting magnets are widely used in laboratories. The most common
form of super conducting magnets is a solenoid made of super conducting wire. The
wonderful thing about a superconducting magnet is that it produces high magnetic
fields without any energy dissipation due to Joule heating, since the electrical
resistance of the superconducting wire becomes zero when the magnet is immersed in
liquid helium at a temperature of 4.2 K. Usually, the magnet is provided with a
specially designed superconducting switch, as shown in Fig. 1. The resistance r of the
switch can be controlled: either r=0 in the superconducting state, or r = rn in the
normal state. When the persistent mode, with a current circulating through the magnet
and superconducting switch indefinitely. The persistent mode allows a steady magnetic
field to be maintained for long periods with the external source cut off.
The details of the superconducting switch are not given in Fig. 1. It is usually a
small length of superconducting wire wrapped with a heater wire and suitably
thermally insulated from the liquid helium bath. On being heated, the temperature of
the superconducting wire increases and it reverts to the resistive normal state. The
typical value of rn is a few ohms. Here we assume it to be 5 . The inductance of a
superconducting magnet depends on its size; assume it be 10 H for the magnet in Fig. 1.
The total current I can be changed by adjusting the resistance R.
This problem will be graded by the plots only!
The arrows denote the positive direction of I, I1 and I2.

Fig. 1
1) If the total current I and the resistance r of the superconducting switch are controlled
10

to vary with time in the way shown in Figs, 2a and 2b respectively, and assuming
the currents I1 and I2 flowing through the magnet and the switch respectively are
equal at the beginning (Fig. 2c and Fig. 2d), how do they vary with time from t1 to
t4? Plot your answer in Fig. 2c and Fig. 2d
Fig.2a

2b

2c

2d

2) Suppose the power switch K is turned on at time t=0 when r=0, I1=0 and R=7.5,
and the total current I is 0.5A. With K kept closed, the resistance r of the
superconducting switch is varied in he way shown in Fig. 3b. Plot the
corresponding time dependences of I, I1 and I2 in Figs. 3a, 3c and 3d respectively.

Fig. 3a

3b

3c

3d

3) Only small currents, less than 0.5A, are allowed to flow through the

11

superconducting switch when it is in the normal state, with larger currents the
switch will be burnt out. Suppose the superconducting magnet is operated in a
persistent mode, i. e. I=0, and I1=i1(e. g. 20A), I2=-i1, as shown in Fig. 4, from t=0
to t=3min. If the experiment is to be stopped by reducting the current through the
magnet to zero, how would you do it? This has to be done in several operation steps.
Plot the corresponding changes of I, r, I1 and I2 in Fig. 4

Fig. 4a

4b

4c

4d

4) Suppose the magnet is operated in a persistent mode with a persistent current of 20A
(t=0 to t=3min. See Fig. 5). How would you change it to a persistent mode with a
current of 30a? plot your answer in Fig. 5.
Fig. 5a

5b

5c

12

5d

Theoretical Problem 2Solution


1) For t=t1 to t3
Since r = 0 , the voltage across the magnet VM = LdI1 / dt =0, therefore,
1
I0 ;
2
1
I 2 = I I1 = I I 0 .
2
I 1 = I 1 (t1 ) =

For t=t3 to t4
Since I2=0 at t=t3, and I is kept at

1
I 0 after
2

t = t 3 , VM = I 2 rn = 0 , therefore, I 1 and I 2 will not change.


I1 =

1
I0 ;
2

I2 = 0
These results are shown in Fig. 6.

Fig. 6a

6b

6c

6d

13

2) For t = 0 to t = 1 min:
Since r = 0 , VM = LdI 1 / dt = 0

I 1 = I 1 (0) = 0
I 2 = I I 1 = 0.5 A.
At t = 1 min, r suddenly jumps from O to rn , I will drop from E / R to
E /( R + rn ) instantaneously, because I 1 can not change abruptly due to the
inductance of the magnet coil. For E / R =0.5A, R = 7.5 and Rn = 5 . I will drop
to 0.3A.
For t = 1 min to 2 min:
I , I 1 and I 2 gradually approach their steady values:
I=

E
= 0.5 A,
R

I 1 = I = 0.5 A
I2 = 0 .
The time constant

L( R + rn )
.
Rrn

When L = 10 H, R = 7.5 and rn = 5 , = 3 sec.


For t = 2 min to 3 min:
Since r = 0 , I 1 and I 2 will not change, that is

I 1 = 0.5 A and I 2 = 0

Fig. 7a

7b

14

7c

7d

3) The operation steps are:


First step
Turn on power switch K, and increase the total current I to 20 A, i. e. equal to I 1 .
Since the superconducting switch is in the state r = 0 , so that VM = L dI 1 / dt = 0 ,
that is, I 1 can not change and I 2 increases by 20A, in other words, I 2 changes
from 20 A to zero.
Second step
Switch r from 0 to rn .
Third step
Gradually reduce I to zero while keeping I 2 < 0.5 A: since I 2 = VM / rn and
Vm = L dI 1 / dt , when L = 10 H, rn = 5 , the requirement I 2 < 0.5 A corresponds to
dI 1 / dt < 0.25 A/sec, that is, a drop of <15A in 1 min. In Fig. 8 dI / dt 0.1A/sec and
dI 1 / dt is around this value too, so the requirement has been fulfilled.
Final step
Switch r to zero when VM = 0 and turn off the power switch K. These results
are shown in Fig. 8.
Fig. 8a

8b

8c

15

8d

4) First step and second step are the same as that in part 3, resulting in I 2 = 0 .
Third step Increase I by 10 A to 30 A with a rate subject to the requirement

I 2 < 0.5 A.
Fourth step Switch r to zero when VM = 0 .
Fifth step Reduce I to zero, I 1 = 30 A will not change because VM is zero.

I 2 = I I 1 will change to 30 A. The current flowing through the magnet is thus


closed by the superconducting switch.
Final step Turn off the power switch K. The magnet is operating in the persistent
mode.
These results are shown in Fig. 9.
Fig. 9a

9b

9c

9d

Grading Scheme
Part 1,

2 points:

0.5 point for each of I 1 , I 2 from t = t1 to t 3 and I 1 , I 2 from t = t 3 to t 4 .


Part 2,

3 points:

0.3 point for each of I 1 , I 2 from t = 0 to 1 min, I , I 1 , I 2 at t = 1 min,

16

and I 0 , I 1 , I 2 from t = 1 to 2 min;


0.2 point for each of I , I 1 , and I 2 from t = 2 to 3 min.
Part 3,

2 points:

0.25 point for each section in Fig. 8 from t = 3 to 9 min, 8 sections in total.
Part 4,

3 points:
0.25 point for each section in Fig. 9 from t = 3 to 12 min, 12 sections in total.

17

Theoretical Problem 3
COLLISION OF DISCS WITH SURFACE FRICTION
A homogeneous disc A of mass m and radius RA moves translationally on a smooth
horizontal x-y plane in the x direction with a velocity V (see the figure on the next
page). The center of the disk is at a distance b from the x-axis. It collides with a
stationary homogeneous disc B whose center is initially located at the origin of the
coordinate system. The disc B has the same mass and the same thickness as A, but its
radius is RB. It is assumed that the velocities of the discs at their point of contact, in the
direction perpendicular to the line joining their centers, are equal after the collision. It
is also assumed that the magnitudes of the relative velocities of the discs along the line
joining their centers are the same before and after the collision.
1) For such a collision determine the X and Y components of the velocities of the two

, V AY
, VBX
and VBY
in terms of m , R A , RB ,
discs after the collision, i. e. V AX
V and b .

2) Determine the kinetic energies E A for disc A and E B for disc B after the collision
in terms of m , R A , RB , V and b .

Theoretical Problem 3Solution


1) When disc A collides with disc B, let n be the unit vector along the normal to
the surfaces at the point of contact and t be the tangential unit vector as shown in the
figure. Let be the angle between n and the x axis. Then we have

b = ( R A + RB ) sin
The momentum components of A and B along n and t before collision are:
mV An = mV cos , mVBn = 0 ,
18

mV At = mV sin , mVBt = 0 .
Denote the corresponding momentum components of A and B after collision by
, mVBn
, mV At , and mVBt . Let A and B be the angular velocities of A and
mV An
B about the axes through their centers after collision, and I A and I B be their
corresponding moments of intertia. Then,
1
I A = mR A2 ,
2
The conservation of momentum gives

IB =

1
mRB2
2

+ mVBn
,
mV cos = mV An

(1)

mV sin = mV At + mVtn ,

(2)

The conservation of angular momentum about the axis through O gives


mVb = mV At ( R A + RB ) + I A A + I B B

(3)

The impulse of the friction force exerted on B during collision will cause a
momentum change of mV At along t and produces an angular momentum I B B
simultaneously. They are related by.
mVBt Rb = I B B

(4)

During the collision at the point of contact A and B acquires the same tangential
velocities, so we have
V At A R A = VBt B RB

(5)

It is given that the magnitudes of the relative velocities along the normal direction
of the two discs before and after collision are equal, i. e.
V An
.
V cos = VBn
From Eqs.

1 and 6 we get

19

(6)

= 0,
V An
= V cos .
VBn
From Eqs. 2 to 5, we get
5
V At = V sin ,
6
1
VBt = V sin ,
6

A =

V sin
,
3R A

B =

V sin
.
3R B

The x and y components of the velocities after collision are:


= V An
cos + V At sin =
V Ax

5Vb 2
,
6( R A + RB ) 2

= V An
sin + V At cos =
V Ay

5Vb ( R A + RB ) 2 b 2
6( R A + RB ) 2

(7)

(8)

5b 2
cos + VBt sin = 1
VBx = VBn
,
2
+
6
(
)
R
R
A
B

sin + VBt cos =


VBy = VBn

5Vb ( RA + RB ) 2 b 2
,
6( RA + RB ) 2

(9)

(10)

2) After the collision, the kinetic energy of disc A is


E A =

3mV 2 b 2
1
1
2 + V Ay
2 ) + I A A2 =
m(V Ax
2
2
8( R A + RB ) 2

(11)

while the kinetic energy of disc B is

E B =

1
1
1
11b 2
2
2
2
2

m(VBx + VBy ) + I B B = mV 1
2
2
2
2
12( R A + RB )

(12)

Grading Scheme
1. After the collision, the velocity components of discs A and B are shown in Eq. 7,
8, 9 and 10 of the solution respectively. The total points of this part is 8. 0. If the result
in which all four velocity components are correct has not been obtained, the point is
marked according to the following rules.
20

0.8 point for each correct velocity component;


0.8 point for the correct description of that the magnitudes of the relative velocities
of the discs along the line joining their centers are the same before and after the
collision.
0.8 point for the correct description of the conservation for angular momentum;
0.8 point for the correct description of the equal tangential velocity at the touching
point;
0.8 point for the correct description of the relation between the impulse and the
moment of the impulse.
2. After the collision, the kinetic energies of disc A and disc B are shown in Eqs.
11 and 12 of the solution respectively.
1.0 point for the correct kinetic energies of disc A;
1.0 point for the correct kinetic energies of disc B;
The total points of this part is 2.0

21

XXV INTERNATIONAL PHYSICS OLYMPIAD


BEIJING, P EOPLES REPUBLIC OF CHINA
PRACTICAL COMPETITION
July 15, 1994
Time available: 2.5 hours
READ THIS FIRST!
INSTRUCTIONS:
1. Use only the ball pen provided.
2. Your graphs should be drawn on the answer sheets attached to the problem.
3. Write your solution on the marked side of the paper only.
4. The draft papers are provided for doing numerical calculations and draft drawings.
5. Write at the top of every page:
The number of the problem
The number of the page of your report in each problem
The total number of pages in your report to the problem
Your name and code number

22

EXPERIMENTAL PROBLEM 1
Determination of light reflectivity of a transparent dielectric surface.
Experimental Apparatus
1. He-Ne Laser(1.5mW).The light from this laser is not linearly polarized.
2. Two polarizers (P1, P2) with degree scale disk (Fig. 1), one (P1) has been
mounted in front of the laser output window as a polarizer, and another one can be
fixed in a proper place of the drawing board by push-pins when it is necessary.
3. Two light intensity detectors (D1, D2) which consisted of a photocell and a
microammeter (Fig. 2).
4. Glass beam splitter(B).
5. Transparent dielectric plate, whose reflectivity and refractive index are to be
determined.
6. Sample table mounted on a semicircular degree scale plate with a coaxial swivel
arm(Fig. 3).
7. Several push-.pins for fixing the sample table on the drawing board and as its
rotation axis.
8. Slit aperture and viewing screen for adjusting the laser beam in the horizontal
direction and for alignment of optical elements.
9. Lute for adhere of optical elements in a fixed place.
10. Wooden drawing board.
11. Plotting papers
Experiment Requirement
1. Determine the reflectivity of the p-component as a function of the incident angle
(the electric field component, parallel to the plane of incidence is called the
p-component).
(a) Specify the transmission axis of the polarizer (A) by the position of the marked
line on the degree scale disk in the p-componet measurement(the transmission
axis is the direction of vibration of the electric field vector of the transmitted
light).
(b) Choose any one of the light intensity detector and set its micro-ammeter at the
range of "5". Verify the linear relation ship between

the light

intensity and

the micro-ammeter reading. Draw the optical schematic diagram. Show your
measured data and

calculated

results(including

the farm of a table. Plot the linear relationship curve.

23

the calculation formula)in

(c) Determine the reflectivity of the p-component as a function of the incident


angle. Draw the optical schematic diagram. Show your measured data and
calculated reflectivity(including the calculation formula)in the form of a table.
Plot the reflectivity as a function of the incident angle.
2. Determine the refractive index of the sample as accurate as possible.
Explanation and Suggestion
1. Laser radiation avoid direct eye exposure.
2. Since the output power of the laser beam may fluctuate from time to time, the
fluctuation of light output has to be monitored during the performance of the
experiment and a correction of the experimental results has to be made.
3. The laser should be lighting all the time, even when you finish your experiment
and leave the examination hall, the laser should be keeping in work.
4. The reflected light is totally plane polarized at an incident angle B while
tg B = n (refractive index).

Fig. 1 polarizers with degree scale disk

Fig. 2

Light intensity detector


24

(1) Insert the plug of photocell into the INPUT socket of microammeter
(2) Switching on the microammeter.
(3) Blocd off the light entrance hole in front of the photocell and adjust the scale
reading of micro ammeter to 0.
(4) Set the Multiple knob to a proper range.

Fig.3 Sample table mounted on a semicircular degree scale plate


Experimental Problem 1Solution
1. (a) Determine the transmission axis of the polarizer and the Brewster angle B of
the sample by using the fact that the rerlectivity of the p-component R p = 0 at
the Brewster angle.
Change the orientation of the transmission axis of P1 , specified by the position of
the marked line on the degree scale disk ( ) and the incident angle ( i ) successively
until the related intensity I r = 0 .

Now the incident light consists of p-component only and the incident angle is B , the

25

corresponding values 1 and B are shown below:

140.0

322.0

141.0

322.5

56.4

56.4

56.2

56.2

1 = 140.5 0.5

or 322.30.1

The Brewster angle B is 56.30.1


1. (b) Verification of the linear relationship between the light intensity and the
microammenter reading.

The intensity the transmitted light passing through two polarized P1 and P2
obeys Malus law
I ( ) = I 0 cos 2
where I 0 is the intensity of the light polarized by p1 and incident, I is the
intensity of the transmitted light, and is the angle between the transmission axes of

P1 and p 2 . Thus we can obtain light with various intensities for the verification by
using two polarizers.
The experimental arrangement is shown in the figure.
The light intensity detector D1 serves to monitor the intensity fluctuation of the
incident beam (the ratio of I 1 to I 2 remain unchanged), and D2 measures I 2 . Let

i1 ( ) and i2 ( ) be the readings of D1 and D2 respectively, and 2 ( ) be the


reading of the marked line position. i2 = 0 when = 90 , the corresponding 2
is 2 (90), and the value of corresponding to 2 is

=| 2 2 (90) 90 |
Data and results;
26

2 (90) = 4

94.0

64.0

49.0

34.0

4.0

0.0

30.0

45.0

60.0

90.0

i1 ( ) A

6.31

5.71

5.71

5.71

5.71

i2 ( ) A

18.75

12.75

8.25

4.05

0.05

From the above data we can obtain the values of I ( ) / I 2 ( ) from the formula
I ( ) i2 ( ) i1 (0)
=

I0
i1 ( ) i2 (0)
and compare them with cos 2 for examining the linear relationship. The results
obtained are:

0.0

30.0

45.0

60.0

90.0

cos 2

1.00

0.75

0.50

0.25

0.00

I ( ) / I 0

1.00

0.75

0.49

0.24

0.00

1. (c) Reflectivity measurement


The experimental arrangement shown below is used to determine the ratio of I 0
to I 1 which is proportional to the ratio of the reading (i20 ) of D2 to the
corresponding reading (i10 ) of D1 .

27

Then used the experimental arrangement shown below to measure the relativity
R p of the sample at various incident angle ( ) while the incident light consists of

p-component only. Let i1 ( ) and i2 ( ) be the readings of

D1 and

respectively.

Then the reflectivity is

R p ( ) =

I ( ) i2 ( ) i10
=

i1 ( ) i20
I0

Data and results:

1 = 140.5
i20 = 19.8 5A
i10 = 13.3A

()

i2 ( )

i1 ( A)

R p ( )

15.10.2

11.1

0.037

10

14.90.2

11.2

0.036

20

13.30.2

11.1

0.032

30

11.40.2

12.2

0.025

40

7.80.2

14.7

0.014

50

2.30.2

16.9

0.0037

53

0.70.2

11.3

0.0017

55

0.30.2

11.3

0.00059

56.3dark

11.5

58

0.30.2

11.5

0.0007

60

1.10.2

13.5

0.0024

64

6.50.2

16.7

0.011

66

7.80.2

11.8

0.018

68

16.30.2

15.0

0.029

28

D2

72

5.30.1

11.7

0.061

76

13.11

14.0

0.13

80

4.45

11.7

0.25

84

9.15

14.5

0.42

The curve of reflectivity of p-component as a function of incident in plexiglass

2. The Brewster angle B can be found from the above date as

B = 56.3 0.2
The index of refraction can be calculated as

n = tan B = 1.50 0.01


The sources of errors are:
1. Detector sensitivity is low.
2. The incident light does not consist of p-component only.
3. The degree scales are not uniform.
EXPERIMENTAL PROBLEM 1: Grading Scheme(10 points)
Part 1. Reflectivity of the p-component. 7 points, distributed as follows.
a. Determination of the transmission axis of the polarizer (A) in p-component
measurement, 1 point.
(Error less than 2,

1.0point;

error less than 3,

0.7point;

error less than 4,

0.3point;

error less than 5

0.1 point.)

b. Verification of the linearity of the light intensity detector(2 points). Draws the
optical schematic diagram correctly, 1.0 point; (Without the correction of the
fluctuation of the light intensity, 0.4 point only);

29

Uses I / I 0 cos 2 figure to show the linearity, 0.5 point;


Tabulate the measured data(with 5 points at least)correctly, 0.5 point.
c. Determination of the reflectivity of the p-component of the light as a function of
incident angle, 4 points, distributed as follows.
Draws the optical schematic diagram correctly and tabulate the measured data
perfectly, 2.0 points;
Plot the reflectivity as the function of incident angle with indication of errors, 2
points.
Part 2. Determination of the refractive index of sample, 3 point.
Brewster angle of sample, 1 point;
(Error less than 1,

1.0point;

error less than 2,

0.5point;

error less than 3,

0.2point;

error larger than 3,

0 point.)

The refractive index of sample, 0.5 point.


Discussion and determination of errors, 1.5 points.
EXPERIMENTAL PROBLEM 2
Black Box
Given a black box with two similar terminals. There are no more than three passive
elements inside the black box. Find the values of elements in the equivalent circuit
between the terminals. This box is not allowed to be opened.
Experimental Apparatus
1. Double channel oscilloscope with a panel illustration, showing the name and
function of each knob
2. Audio frequency signal generator with a panel illustration, showing the name
and function of each knob
3. Resistance box with a fixed value of l00 ohm(< 0.5%)
4. Several connecting wires
5. For the coaxial cables,

the wire in black color at the terminal is grounded.

6. Log-log paper, semi-log paper, and millimeter paper are provided for use if
necessary
Note: The knobs, which were not shown on the panel illustration of the signal
generator and oscilloscope, have been set to the correct positions. It should
not be touched by the student.
Experimental Requirements
1. Draw the circuit diagram in your experiment.
30

2. Show your measured data and the calculated results in the form of tables. Plot
the experimental curves with the obtained results on the coordinate charts
provided(indicate the title of the diagram and the titles and scale units of the
coordinate axes)
3. Given the equivalent circuit of the black box and the names of the elements with
their values in the equivalent circuit(write down the calculation formulas).
Instructions
1. Do your experiment in the frequency range between 100 Hz and 50kHz.
2. The output voltage of the signal generator should be less than 1.0V
(peak-to-peak). Set the Out Attenuation switch to 20 db position and it
should not be changed.
3. On connecting the wires, be careful to manage the wiring so as to minimize the
50Hz interference from the electric mains.
Instruction for Using XD2 Type Frequency Generator
1. Set the Out Attenuation to 20 db position and it should not be changed.
2. Set the Damping Switch to Fast position.
3. The indication of the voltmeter of the signal generator is the relative value, but
not the true value of the output.
4. Neglect the error of the frequency readings.
Note: For XD22 Type Audio Frequency generator, there is no Damping Switch, and
the output switch should be set to the sine ~ position.
Instruction for Using SS-5702 Type Oscilloscope
1. Keep the V mode switch in Dual position.
2. The Volts/div (black) and the variable control (red) vary the gain of the
vertical amplifier, and when the variable control (red) is ill the fully
clockwise position, the black setting are calibrated.
3. The Times/div (Black) varies the horizontal sweep rate from 0.5s/div to
0.2s/div, and they are calibrated when the variable control (red) is in the fully
clockwise CAL position.
4. The Trigging Source (Trigging sweep signal) is used to select the trigging
signal channel and the" level" control is used to adjust the amplitude of the
trigging signal.
5. Measuring accuracy: 4%.
Instruction for Using Resistance Box
The resistance of the Resistance Box has been set to a value of 100ohm, and it
should not be changed.
31

Experimental problem 2......

Solution

1. The circuit diagram is shown in Fig. 1

Fig. 1
We have the relation:
I=

Z+R=

VR
;
R

VZ + R VZ + R
=
R
I
VR

2. Measure the values of VZ + R and VR at various frequencies (f), the measured data
and calculated value of Z+R are shown in table l. The Z+R-f curve is plotted in Fig. 2

Table l. The magnitude of impedance verus frequency


f (10 3 Hz)

U Z + R (V pp )

U R mVpp

Z + R (10 3 )

0.100

0.600

22.0

2.73

0.200

0.600

45.0

1.33

0.400

0.600

94.0

0.638

0.700

0.300

92.0

0.326

0.900

0.300

121

0.248

32

1.00

0.300

136

0.220

1.10

0.300

140

0.214

1.16

0.300

141

0.213

1.25

0.300

140

0.214

1.50

0.300

120

0.250

2.00

0.300

88.0

0.341

4.00

0.300

78.0

0.769

8.00

0.600

38.0

1.58

15.0

0.600

20.0

3.00

30.0

0.600

10.0

6.00

50.0

0.600

6.0

10.0

From table 1 and Fig. 2, we got the conclusions:


(1) Current resonance (minimum of Z) occurs at f 0 1.16 10 3 Hz.
(2) f f 0 , Z f , / 2 . The impedance of the black box at low
frequency is dominated by a inductance.
(3) f f 0 , Z f , / 2 . The impedance of the black box at high
frequency is dominated by a inductance.
(4) Equivalent circuit of the black box; r, L and C connected in series shown in
Fig. 3.

Fig. 3
3. Determination of the values of r , L and C .
(a) r
At resonance frequency f 0

33

VC = VL
Then

Z+R=

VZ + R VZ + R
=
R=r+R
I
VR

From table 1, r + R = 213 , it is given R = 100 , so the equivalent resistance


r in Fig. 3 is equal 113 .

(b) C
At low frequency, z L 0 in Fig. 3. So the circuit could be considered as a series
RC circuit.
From phasor diagram, Fig. 4,
VZ2+ R VR2+ r
V
1
= ZC = C =
C
I
I

Since VR2+ r / VZ2+ R 6 10 3 at f = 100 Hz, V R2+ r can beneglected with respect to
VZ2+ R , so
V
1
Z + R Z + R = 2.73 10 3
I
C

1
= 0.58f .
( Z + R)

C 0.58f .

Fig. 4

(c) L
At high frequency, Z L 0 in Fig. 3. So the circuit could be considered as a series
RL circuit.
From phasor diagram, Fig. 5,
| VL |= VZ2+ R Vr2+ R ,

Since Vr2+ R / VZ2+ R 4.5 10 4 at f = 50 kHz, Vr2+ R can be


neglected with respect to VZ2+ R , so

L = Z L =

Fig. 5
V L | VZ + R |
=
Z + R = 10 4
I
I

34

(3)

L=

Z+R

= 31.8 mH.

Error estimation:
It is given, precision of the resistance box reading R / R 0.5%
precision of the voltmeter reading V / V 4%
(1) Resistance r : at resonance frequency f 0

r+R=

VZ + R
R
VR

(r + R) VZ + R VR R
=
+
+
4% +4%+0.5%=8.5%
VR
R
VZ + R
r+R
r = 16

(2) Capacitance C: (Neglect the error of the frequency reading)


V
1
Z C = Z +R R
VR
C
C VZ + R VR R
=
+
+
8.8%
C
VZ + R
VR
R
The approximation VC VZ + R will introduce apercentageerror 0.3%
(3) Inductance L: Similar to the results of capacitance C, but the percentage error
introduced by the approximation VL VZ + R is much small (0.003%) and thus
negligible.
L
8.5% .
L

Experimental Problem 2: Grading Scheme (10 points maximum)


1. Measuring circuit is correct as shown in Fig.(a)
2.0point

Fig. a
35

2. Correct data table and figure to show the characteristic of the black box
2.0 points
3. The equivalent circuit of the black box, and the names of the elements with their
values in the equivalent circuit are correct
total 6.0 points
(a) R, L and C are connected in series
1.5 point
(L and C are connected in series
1.0 point)
(b)

Correct value (error less than 15% ) for each element


0.5 point (3)
(error between 15% and 30% 0.3)
(error between 30% and 50% 0.1)

(c) Correct calculation formula for each element


0.5 point (3)
(d)

Error estimate is reasonable for each element


0.5 points (3)

36

Вам также может понравиться